You are on page 1of 59

Glaucoma

Question 1 of 130
One week after a trabeculectomy a patient has a flat peripheral anterior chamber with iris-to-cornea touch, but no lens-to-
cornea touch, applanation tension near 0 mm Hg, large choroidal effusions, and a positive Seidel test with a flat bleb. What
would the single most important step be at this time?

Close the leak

Re-form the anterior chamber

Increase topical corticosteroid therapy

Drain the choroidal effusions


Please select an answer
Feedback: For any patient with a flat bleb and a visible leak, with or without a shallow anterior chamber, measures should
be undertaken to close the leak. If bleb function is not severely compromised, conservative maneuvers, such as pressure
patching and aqueous suppressant therapy, may succeed. However, if the bleb is completely flat with a large leak, it is
usually necessary to close the leak surgically. Glue with a contact lens would be another alternative, but this technique can
compromise bleb function. Simply re-forming the anterior chamber or draining the choroidal effusions will not solve the
important problem of the bleb leak. If there is no lens-to-cornea touch, anterior chamber re-formation may not be necessary.

Question 2 of 130
The administration of topical ocular miotic (cholinergic) therapy in an eye with active intraocular inflammatory disease
(uveitis) will not increase which of the following?

Posterior synechia formation

Uveoscleral outflow

Pain

Inflammation
Please select an answer
Feedback: Topical ocular miotic (cholinergic) therapy increases trabecular outflow, but decreases uveoscleral outflow. In an
eye with active intraocular inflammatory disease, miotic therapy may cause increased pain, inflammation, and posterior
synechia formation. In these eyes, miotic therapy should be avoided. Preferred medical management includes cycloplegic
and corticosteroid therapy and glaucoma therapy other than miotics.
Question 3 of 130
One week after blunt trauma and hyphema, a patient with no previous history of glaucoma has a pressure of 40 mm Hg.
Medical management has included levobunolol (Betagan), dorzolamide (Trusopt), homatropine 5%, and prednisolone
acetate 1% qid. What is the least likely cause of the intraocular pressure elevation?

Blood obstructing the trabecular meshwork

Direct trauma to the trabecular meshwork

Corticosteroid-induced intraocular pressure elevation

Pupillary block
Please select an answer
Feedback: Shortly after blunt trauma, blood and inflammatory debris may clog the trabecular meshwork and raise the
intraocular pressure. Direct injury to the trabecular meshwork can also occur, with angle recession as a sign of that injury. A
large clot can occlude the pupil, and extensive posterior synechia formation can cause a pupillary-block mechanism. Topical
costeroid therapy may help control intraocular pressure by reducing inflammation and preventing the formation of peripheral
anterior and posterior synechiae. A corticosteroid-induced intraocular pressure usually does not occur until at least 2-3
weeks after initiation of corticosteroid therapy. If there is visible blood and inflammatory debris in the anterior chamber,
corticosteroid-induced intraocular pressure elevation should not be assumed to be playing an important role.

Question 4 of 130
A 56-year-old diabetic patient presents with pain and decreased vision to 20/80. Intraocular pressure is 58 mm Hg. There is
mild corneal epithelial edema, iris neovascularization, a mild nuclear sclerotic cataract, and clear vitreous with visible
proliferative diabetic retinopathy in that eye. After initiating medical therapy, what is the most important first step?

Trabeculectomy with intraoperative mitomycin C or postoperative subconjunctival 5-fluorouracil

Placement of a glaucoma implant

Cyclophotocoagulation

Panretinal photocoagulation
Please select an answer
Feedback: In an eye with neovascular glaucoma and proliferative diabetic retinopathy, after medical therapy is initiated, the
most important first step is to perform a heavy panretinal photocoagulation (PRP). If the procedure is performed shortly after
the development of neovascular glaucoma, there may be little permanent synechial angle closure, and the glaucoma may
abate or be controlled medically. Once substantial synechial angle closure occurs, however, other forms of management in
addition to PRP become necessary. If filtering surgery is performed, either as a trabeculectomy with or without antifibrosis
therapy or as a glaucoma implant (seton), there is little chance of success and a high complication rate without some
preceding retinal ablation and regression of rubeosis. In a highly inflamed eye with active neovascularization and acceptable
intraocular pressure, there might be some benefit in delaying filtration surgery until some visible regression of the iris
neovascularization occurs.
Question 5 of 130
Of the following, which would be the best initial medical agent for a patient with severe asthma and newly diagnosed
glaucoma?

Carteolol (Ocupress)

Betaxolol (Betoptic)

Timolol (Timoptic, Betimol)

Dorzolamide (Trusopt)
Please select an answer
Feedback: Nonselective beta-adrenergic antagonist therapy (carteolol, metipranolol, timolol) may precipitate increased
shortness of breath or an asthma attack in patients with asthma. The selective beta-1 adrenergic antagonist betaxolol
reduces the chance of this complication, but does not eliminate it in the severely asthmatic patient. Beta-receptor selectivity
is relative and not absolute. In the patient presented, dorzolamide, a topical carbonic anhydrase inhibitor, would be the
preferred medical option of the four listed. Latanoprost (Xalatan) is also indicated for first-line therapy. Other options are
brimonidine (Alphagan), and less often used, dipivefrin (Propine) and pilocarpine.

Question 6 of 130
In an eye with a narrow angle, which of the following most strongly argues in favor of performing a laser peripheral
iridotomy?

Intraocular pressure level

Amount of glaucomatous optic nerve cupping

Amount of glaucomatous visual field loss

Gonioscopic findings
Please select an answer
Feedback: In chronic angle-closure glaucoma with relative pupillary block, gonioscopy is the key to diagnosis and
management. Intraocular pressure may be normal or elevated. The presence of elevated pressure with a narrow angle
alone does not indicate the need for a laser iridotomy. In this case, an open angle may be causing the intraocular pressure
elevation with a coexistent narrow angle. In addition, the extent of visual field loss or optic nerve damage does not indicate
whether an iridotomy is needed. Patients with appositional angle closure or areas of peripheral anterior synechiae with
relative pupillary block have a high risk for developing chronic angle closure and should have a laser iridotomy.
Question 7 of 130
An elderly patient underwent an intracapsular cataract extraction many years ago. More recently, a full-thickness
sclerectomy was performed in that eye for uncontrolled open-angle glaucoma. Now the patient presents having had pain
and decreased vision for 2 days. Visual acuity is finger counting, the applanation tension is 7 mm Hg, there is no bleb, and
there is a large hypopyon. There is no view of the posterior segment. B-scan ultrasonography demonstrates substantial
vitreous debris. Which of the following would not be an appropriate treatment?

Topical fortified antibiotic therapy alone

Intravitreal antibiotic therapy

Pars plana vitrectomy with intraocular antibiotic injection

Vitreous tap for cultures


Please select an answer
Feedback: In an aphakic eye or a pseudophakic eye with a disrupted or absent posterior capsule, intraocular infection from
a bleb may quickly enter the vitreal cavity. The prognosis for such eyes is much worse than those with a blebitis. To date,
there is no controlled study evaluating the treatment of late endophthalmitis in eyes with filtering blebs that is similar to the
Endophthalmitis Vitrectomy Study (EVS), which evaluated the roles of pars plana vitrectomy and systemic antibiotics in
treating postoperative endophthalmitis. Historical studies and reports have suggested that bleb endophthalmitis is often
caused by Streptococcus pneumoniae or Haemophilus influenzae and, even with aggressive treatment, may have a poor
outcome. At this time, treatment of this patient might include topical fortified antibiotic therapy, intravitreal antibiotic therapy,
and vitrectomy with intraocular antibiotic injection.

Question 8 of 130
A 72-year-old woman presents with bilateral uncontrolled primary angle glaucoma and cataracts. Her best-corrected visual
acuity is 20/40 OD and 20/60 OS. She complains of increasing difficulty driving and reading fine print. Intraocular pressure is
23 mm Hg bilaterally. Her current medications are carteolol (Ocupress) bid OU, latanoprost (Xalatan) qhs OU, and
dorzolamide (Trusopt) tid OU. She has not tolerated pilocarpine in the past, stating it blurred and dimmed her vision. She
has no prior history of ocular laser or surgery. She has moderate visual field loss, with bilateral superior arcuate scotomas.
Which of the following is not a reasonable therapeutic choices at this time?

Laser trabeculoplasty

Trabeculectomy followed by cataract surgery at a later time

Cataract surgery

A combined cataract extraction and trabeculectomy


Please select an answer
Feedback: This individual has uncontrolled, moderately advanced glaucoma in both eyes and visually significant cataracts.
If the patient wishes to have cataract surgery, a combined cataract extraction with trabeculectomy would be a good
therapeutic choice. Trabeculectomy can be performed first, but would delay visual recovery until the time of a cataract
surgery at a later date. If the patient is reluctant to have cataract surgery or glaucoma surgery at this time, laser
trabeculoplasty would be a reasonable choice; however, over the long term, this may not adequately lower intraocular
pressure. Laser trabeculoplasty would have a greater effect if performed before cataract surgery. Cataract surgery alone in
an eye with uncontrolled glaucoma, on three medications, and with moderate visual field loss, would probably be the poorest
therapeutic option among the four.
Question 9 of 130
In which one of the following situations would intraoperative mitomycin C application during trabeculectomy be most clearly
indicated for?

A patient with previous unsuccessful glaucoma filtering surgery

A 25-year-old patient

A patient with traumatic glaucoma with angle recession

A black patient
Please select an answer
Feedback: Antifibrosis agents (5-fluorouracil or mitomycin C) are beneficial when performing glaucoma filtering surgery in a
patient with a history of previous cataract surgery or previous unsuccessful glaucoma filtering surgery. The 5-Fluorouracil
Filtering Surgery Study demonstrated the benefit of postoperative subconjunctival 5-fluorouracil injections in these 2 groups
of patients. Of the 4 choices provided, mitomycin C would be most strongly indicated in the patient with previous
unsuccessful glaucoma filtering surgery. It is also possible that 5-fluorouracil or mitomycin C would benefit a patient with
neovascular glaucoma (assuming panretinal photocoagulation has been performed) or ocular inflammatory disease.
Although the use of mitomycin C would be less critical and possibly risk postoperative hypotony in a young or black patient,
or in an individual with previous blunt trauma, 5-fluorouracil may be appropriate and a safer choice for some of these
patients. Extent of glaucomatous visual loss and the need for a lower postoperative intraocular pressure may also affect the
decision to use mitomycin C, 5-fluorouracil, or no antifibrosis agent.

Question 10 of 130
A patient presents 2 years after a trabeculectomy during which mitomycin C was administered. She has had pain, redness,
and discharge for 1 day in that eye. Examination reveals a visual acuity of 20/200, applanation tension of 4 mm Hg,
mucopurulent discharge, small bleb leak, and extensive cellular reaction in the anterior chamber with a small hypopyon. The
lens and vitreous are clear. What would the most appropriate treatment be at this time?

Intravenous antibiotic therapy

Pars plana vitrectomy with intraocular antibiotic injection

Topical fortified antibiotic therapy

Injection of antibiotics the anterior chamber


Please select an answer
Feedback: In a patient with an infected bleb (blebitis) and anterior chamber reaction, topical fortified broad-spectrum
antibiotic therapy would be the most appropriate initial treatment. In the presence of a clear vitreous, a pars plana vitrectomy
with antibiotic therapy would not be necessary. Injection of antibiotics into the anterior chamber would also not be
necessary. Intravenous antibiotic therapy would be no more effective and could arguably be less effective than topical
fortified broad-spectrum antibiotic therapy. After the organism and its sensitivities have been identified, specific antibiotic
therapy can be continued.
Question 11 of 130
A laser peripheral iridotomy would most likely relieve angle-closure glaucoma in which of the following conditions?

Pseudophakic pupillary block

Angle-closure glaucoma scleritis

Neovascular glaucoma

Iridocorneal endothelial syndrome


Please select an answer
Feedback: A laser peripheral iridotomy is indicated in cases with primary and secondary angle-closure glaucoma with
relative or complete papillary block. This includes eyes with acute, subacute, and chronic closure glaucoma, all with a
relative pupillary block, and in eyes with phakic, aphakic, or pseudophakic pupillary-block glaucoma. Angle-closure
mechanisms that operate by pulling the iris into the angle, such as occurs with neovascular glaucoma or iridocorneal
endothelial syndrome, would not be expected to benefit from a laser peripheral iridotomy. During angle closure from scleritis,
the peripheral iris is anteriorly displaced by rotation of the ciliary body from the thickened and edematous sclera. This
mechanism of glaucoma may respond to topical cycloplegic therapy and systemic corticosteroid therapy; however, if laser
therapy is needed, a peripheral iridoplasty would be more beneficial than an iridotomy.

Question 12 of 130
A patient with no history of glaucoma presents with a very low intraocular pressure after recent cataract surgery. The
surgeon reported that the surgery was uneventful, the posterior capsule was intact, and a posterior chamber lens implant
was inserted in the capsular bag. A limbal incision was constructed superiorly, 1 mm behind the anterior limbal margin, and
closed with 10-0 nylon sutures. What is the least likely cause of a low intraocular pressure in this eye after cataract surgery?

Wound leak

Rhegmatogenous retinal detachment

Inadvertent bleb formation

Cyclodialysis cleft
Please select an answer
Feedback: Low intraocular pressure after cataract surgery is not an uncommon finding. The surgeon must first search for
evidence of a wound leak or rhegmatogenous retinal detachment. If neither of those two conditions is found, the next most
likely reason for postoperative hypotony, in an eye with a incision, is inadvertent filtration, even in the absence of a
substantial filtering bleb. Sometimes these eyes have a small diffuse area of uplifted conjunctiva with microcyst formation. A
cyclodialysis cleft after cataract surgery with a limbal incision would be the least likely of the four listed options. Factors that
would suggest the presence of a dialysis cleft, in addition to finding one on gonioscopy, would be a history of traumatic
surgery with capsular disruption, vitreous loss, iridodialysis, hyphema, or other evidence of difficult cataract surgery.
Cyclodialysis cleft may occur after a deep scleral tunnel incision.
Question 13 of 130
Which of the following glaucomas might not resolve after cataract extraction?

Primary angle-closure glaucoma

Microspherophakia with glaucoma

Phacolytic glaucoma

An eye with angle recession and phacodonesis after blunt trauma


Please select an answer
Feedback: In phacolytic glaucoma, a release of proteinaceous lens material that is engulfed by macrophages clogs the
trabecular meshwork and causes secondary elevation of intraocular pressure. Cataract surgery may cure this glaucoma. In
microspherophakia, pupillary block is induced by the abnormal spherical shape of the lens. A laser peripheral iridotomy or
removal of the lens would be expected to relieve the pupillary block. In chronic primary angle-closure glaucoma, relative
pupillary block is induced by the tension of the iris sphincter muscle against the slightly anteriorly positioned lens in an eye
with a relatively short axial length. Cataract surgery may improve glaucoma control or may completely eliminate glaucoma in
these eyes. Cataract extraction would not be expected to improve pressure control in an eye with blunt trauma and direct
trabecular damage. Angle recession may be present and is evidence of the trabecular damage. Phacodonesis is evidence of
zonular disruption. In this patient, the lens is not contributing to the intraocular pressure elevation.

Question 14 of 130
Which of following would be at greatest risk for delayed choroidal hemorrhage after glaucoma filtering surgery?

A highly myopic eye

An aphakic eye with a concurrent or previous vitrectomy

An elderly patient

A phakic eye after a mitomycin C trabeculectomy


Please select an answer
Feedback: Ocular increases risk of a delayed suprachoroidal hemorrhage after glaucoma filtering surgery. However, two of
the greatest risk factors for delayed suprachoroidal hemorrhage after glaucoma filtering surgery are surgical aphakia and a
history of previous or concurrent operative vitrectomy. Absence of lens and vitreous have been hypothesized to increase the
risk of development and expansion of choroidal hemorrhage. A phakic eye with hypotony after a mitomycin C
trabeculectomy would be at risk for development of effusions and/or hypotony maculopathy. The risk of choroidal
hemorrhage in this patient would be less than in a patient with an aphakic, vitrectomized eye. Elderly patients may be more
likely to develop choroidal hemorrhage than younger patients, and highly myopic patients more likely than emmetropic
patients. However, aphakia and previous or intraoperative vitrectomy are more significant risk factors.
Question 15 of 130
Which of the following risk factors is probably the least significant for primary open-angle glaucoma?

Race or ethnic origin

Family history

Myopia

Level of intraocular pressure


Please select an answer
Feedback: Although glaucoma is not equated with elevated intraocular pressure, intraocular pressure is probably the most
important risk factor for visual loss. Family history and racial or ethnic background are also highly important risk factors.
Black Americans have a greater prevalence of open-angle glaucoma at all ages. There is mounting evidence for the genetic
basis of many of the glaucomas. Other factors, such as myopia and diabetes mellitus, are probably less important risk
factors for primary open-angle glaucoma.

Question 16 of 130
Which of the following is not associated with normal-tension (low-tension) glaucoma?

Migraine

Small optic nerve

Systemic vascular disease

Disc (Drance) hemorrhages


Please select an answer
Feedback: In normal-tension glaucoma, there is evidence of active glaucoma progression despite intraocular pressure
levels that are normal or marginal. Normal-tension glaucoma is more commonly found among patients with known migraine
and/or vascular disease. Drance hemorrhages, suggestive of active glaucoma progression, can be found both in high-
tension and normal-tension glaucoma, but are seen more often in normal-tension glaucoma. There is a suggestion that
larger optic nerves, such as those sometimes seen in high myopes, may be more susceptible to damage from elevated or
marginal intraocular pressure. Small optic nerves with small cups tend to be more resistant to glaucomatous damage from
elevated intraocular pressure, but are more susceptible to nonarteritic ischemic optic neuropathy.
Question 17 of 130
In an eye with increased episcleral venous pressure, which of the following medical agents would be expected to have the
least intraocular pressure-lowering effect?

Topical miotic (cholinergic) therapy (pilocarpine, carbachol, echothiophate iodide)

Beta-adrenergic antagonist (betaxolol, carteolol, levobunolol, metipranolol, timolol)

Topical or oral carbonic anhydrase inhibitor (acetazolamide, methazolamide, dichlorphenamide, dorzoloamide)

Topical prostaglandin agent (latanoprost)


Please select an answer
Feedback: Eyes with increased episcleral venous pressure respond poorly to medications that increase trabecular outflow.
Because of this, these eyes tend to respond poorly to topical miotic therapy or laser trabeculoplasty. Medical agents that
reduce aqueous production (beta-adrenergic antagonists, carbonic anhydrase inhibitors) or that increase nontrabecular
outflow (prostaglandins) would be expected to have a greater effect.

Question 18 of 130
Which of the following was not demonstrated by the Glaucoma Laser Trial?

Laser-first-treated eyes had a slightly better visual field score than medication-first-treated eyes.

Laser-first-treated eyes had a slightly lower intraocular pressure than medication-first-treated eyes.

Most eyes in the laser-first group did not require medical therapy to control pressure.

Laser trabeculoplasty is about as effective as one glaucoma medical agent, timolol (Timoptic).
Please select an answer
Feedback: The Laser Trial has demonstrated that laser trabeculoplasty is a safe and reasonable alternative to medication
(tirnolol) in the initial treatment of open-angle glaucoma. However, 2 years later, approximately 56% of laser-first-treated
eyes needed medication to adequately control pressure. Laser-first-treated-eyes had both a lower mean intraocular pressure
and a higher mean visual field score after treatment than the medication-first-treated eyes. Some of the difference in visual
field scores may be explained by the larger mean pupil size for the laser-first-treated eyes.

Question 19 of 130
Important evidence of progression of primary infantile glaucoma in an infant would not include which of the following?

Increasing axial length

Amblyopia

Increasing horizontal diameter

Increasing myopia
Please select an answer
Feedback: In glaucomas with an onset later than early childhood, the most important evidence of progression is found by
examining the optic nerve and the visual field. However, in infants, uncontrolled intraocular pressure causes a diffuse
enlargement of the globe manifested by increasing myopia, horizontal corneal diameter, and axial length. Amblyopia can be
a substantial problem, even when infantile glaucoma is well controlled, due to significant anisometropia, uncorrected
astigmatism, or early visual deprivation.

Question 20 of 130

Automated visual field testing does not have which advantage listed below?

More patient-tester interaction

Less training required for the tester (technician) than is the case with Goldmann visual field testing

Greater standardization

Ability to compare statistically the current visual field with previous visual field tests

Please select an answer

Feedback: Automated threshold visual field testing is more standardized, allows comparison of current to previous visual
field tests with statistical analyses, and requires less technical training for the person performing the visual field test than is
the case with Goldmann manual perimetry. However, greater patient-tester interaction occurs with manual Goldmann
perimetry than with automated perimetry. In automated perimetry, a computerized program presents the visual field test.
Good patient preparation and instructions will, however, help produce a better quality automated visual field test. Technician
observation and encouragement during the test can improve the quality of an automated visual field test. New
developments, including short wavelength automated perimetry, will increase the usefulness of this test.

Question 21 of 130
Assuming equal transmission and absorption of laser energy, which time and power setting below would provide energy
equal to that delivered by an argon laser with settings of 0.1 sec duration and 500 mW power?

0.02 sec; 2.5 W

1 sec; 5 W

0.02 sec; 1 W

0.05 sec; 2 W
Please select an answer
Feedback: The energy delivered by the argon laser in a "perfect" setting is determined by the power in watts or milliwatts,
multiplied by the duration in seconds. The equation for this relationship is power x time = energy. If the power is increased
and the time is proportionately decreased, the same amount of energy will be delivered. 0.02 sec; 2.5 W is the only
response demonstrating that relationship. Both these parameters and those provided in the question deliver 50 millijoules of
energy.
Question 22 of 130
Medical management of glaucoma associated with inflammatory ocular disease (uveitis) and active intraocular inflammation
would not include which of the following?

Beta blocker

Pilocarpine

Cholinergic antagonist (cycloplegic agent)

Topical corticosteroid
Please select an answer
Feedback: In a patient with active intraocular inflammation, topical corticosteroid and cycloplegic therapy is appropriate. A
topical beta blocker will help reduce intraocular pressure by decreasing aqueous production. Pilocarpine should not be used
because it will increase pain and inflammation and may lead to posterior synechiae formation and a poorly dilating pupil,
which could progress to pupillary-block glaucoma.

Question 23 of 130
Which of the following would be the weakest indication for a combined cataract extraction and trabeculectomy in a patient
with glaucoma and a visually significant cataract?

Glaucoma controlled with one medication in an eye with advanced glaucomatous visual field loss

An eye with previous trabeculectomy and with intraocular pressure of 18 mmHg on a beta blocker and miotic

An eye with a previous history of acute angle-closure glaucoma, treated with laser iridotomy, and now with an
intraocular pressure of 17 mmHg on no medication and with no peripheral anterior synechiae

Well-controlled glaucoma (intraocular pressure 13 mmHg) on a topical beta blocker and miotic agent, and oral carbonic
anhydrase inhibitor
Please select an answer
Feedback: A combined cataract extraction with trabeculectomy can be performed in an eye with well-controlled glaucoma
on multiple antiglaucoma medications. A combined procedure is also advantageous for the patient with advanced
glaucomatous visual loss, who might suffer further visual loss, including loss of fixation, in the immediate postcataract
surgical period if there is substantial intraocular pressure elevation. Even in the eye with a previous trabeculectomy, a
combined procedure can be helpful if the filter's function is not particularly good or not expected to survive cataract surgery.
The patient with normal intraocular pressure on no medication and without substantial peripheral anterior synechiae after an
episode of acute angle-closure glaucoma would be the least likely of this group to require a combined procedure.

Question 24 of 130
What is the most common reason for long-term visual loss in primary infantile glaucoma?

Corneal edema

Amblyopia

Corneal scarring

Glaucomatous optic nerve damage


Please select an answer
Feedback: Amblyopia is the most common cause of long-term visual loss in eyes with primary infantile glaucoma. Corneal
edema often resolves after a surgical procedure to reduce intraocular pressure. Breaks in Descemet's membrane and mild
corneal scarring can occur but usually do not cause substantial visual loss. Serious visual loss from glaucomatous optic
nerve damage can occur, but it is less common than amblyopia.

Question 25 of 130
A patient presents 2 years after glaucoma filtering surgery with purulent discharge and endophthalmitis. Which of the
following is the most likely causative organism?

Staphylococcus epidermidis

Streptococcus pneumoniae

Pseudomonas aeruginosa

Propionibacterium acnes
Please select an answer
Feedback: A late bleb-associated endophthalmitis tends to be caused by Streptococcus pneumoniae (pneumococcus) or
Haemophilus influenzae. Staphylococcus aureus and Staphylococcus epidermidis are more commonly associated with
early-onset endophthalmitis after cataract surgery. Propionibacterium aenes has been assocated with a later-onset
endophthalmitis after cataract surgery. Pseudomonas aeruginosa causes a fulminant endophthalmitis but is not frequently
reported as a causative agent of late bleb-associated endophthalmitis.

Question 26 of 130
Which one of the following does not cause a superior visual field defect in automated threshold perimetry?

Glaucoma

Lens rim artifact

High false-positive rate

Ptosis
Please select an answer
Feedback: Glaucoma, ptosis, and lens rim artifact can cause superior visual field defects. A high false-positive rate, which
indicates that the patient responded when no stimulus was presented, may be due to a nervous patient and usually causes
increased thresholds rather than a visual field defect in automated threshold perimetry.
Question 27 of 130
Which of the following is not a risk factor for failure after glaucoma filtering surgery?

Aphakia

Iris neovascularization

Uveitis

Pigmentary dispersion
Please select an answer
Feedback: Aphakia, uveitis, and iris neovascularizationare are risk factors for failure of standard glaucoma filtering surgery.
Pigmentary dispersion alone is not a risk factor for failure. However, patients with pigmentary dispersion and pigmentary
glaucoma tend to be younger, which may represent a risk factor for failure.

Question 28 of 130
A 45-year-old black woman with a history of sarcoidosis presents with pain, decreased vision, and elevated intraocular
pressure in one eye. Examination reveals 1+ cell and flare and an intraocular pressure of 32 mmHg. She was last seen 9
days earlier with similar complaints and was placed on prednisolone acetate 1%, 4 times daily, and a topical ocular beta-
adrenergic antagonist 2 times daily. She has requested another opinion. Gonioscopy demonstrates light, irregular trabecular
pigmentation and a few scattered peripheral anterior synechiae, but the angle is predominantly open. What would the best
next treatment step be?

Perform laser trabeculoplasty

Discontinue the topical corticosteroid

Add pilocarpine therapy

Increase the frequency of the topical corticosteroid


Please select an answer
Feedback: In a patient with intraocular inflammatory disease (uveitis) and elevated intraocular pressure, it is essential to
reduce intraocular inflammation to prevent the development of peripheral anterior synechiae, posterior synechiae, and other
intraocular from inflammation. The addition of pilocarpine would not be indicated because it would increase intraocular
inflammation and pain and encourage the formation of posterior and peripheral anterior synechiae. Laser trabeculoplasty is
contraindicated in eyes with active intraocular inflammatory disease. It would not lower the intraocular pressure and could
increase intraocular pressure, and the formation of peripheral anterior synechiae. Because there is active intraocular
inflammation and because corticosteroid therapy has been used for only 2 weeks, this should not be discontinued. A
corticosteroid-induced pressure elevation is unlikely at this early time, but could be considered later. The best treatment step
for this patient would be to substantially increase the frequency of topical corticosteroid therapy to attempt to reduce, if not
eliminate, active intraocular inflammation. This may lead to a normalization of intraocular pressure. If the intraocular
pressure is not reduced, then other antiglaucoma medical therapy can be instituted, including an oral or topical carbonic
anhydrase inhibitor, apraclonidine, or brimonidine, or, if necessary, glaucoma filtering surgery can be performed.
Question 29 of 130
Which of the following is not commonly seen in primary infantile glaucoma?

Prominent, anteriorly displaced Schwalbe's line

Breaks in Descemet's membrane

Myopia

Increased corneal diameter


Please select an answer
Feedback: Elevated intraocular pressure in primary infantile glaucoma causes a generalized enlargement of the globe. This
can be manifested as an increased corneal diameter, progressive myopia, and breaks in Descemet's membrane. The
anterior chamber angle is usually malformed, with a high iris insertion and the appearance of a membrane-like structure over
the trabecular meshwork. A prominent, anteriorly displaced Schwalbe's line, known as posterior embryotoxon, can be seen
in Axenfeld-Rieger syndrome but is not typically associated with primary infantile glaucoma.

Question 30 of 130

Which of the following would be the weakest indication for antifibrotic therapy in conjunction with glaucoma filtering surgery?

Glaucoma in pseudophakia

Primary trabeculectomy and exfoliation syndrome (pseudoexfoliation) glaucoma

Neovascular glaucoma

Previously failed glaucoma filtering surgery

Please select an answer

Feedback: The Fluorouracil Filtering Surgery Study (FFSS) demonstrated the value of postoperative subconjunctival 5-
fluorouracil in patients undergoing trabeculectomy after previously failed glaucoma filtering surgery and in aphakic or
pseudophakic eyes. A number of nonrandomized reports have suggested that 5-fluorouracil also may be beneficial in eyes
with neovascular glaucoma. However, filtering surgery with antifibrotic therapy has little chance of success in an eye with
neovascular glaucoma unless panretinal laser photocoagulation has been performed and there has been at least some
regression of the iris neovascularization. Intraoperative mitornycin-C is an alternative to 5-fluorouracil. Eyes with exfoliation
syndrome glaucoma are typically not at a higher risk for failure after primary glaucoma filtering surgery.
Question 31 of 130
Which of the following is not associated with chronic angle-closure glaucoma with relative pupillary block?

Hyperopia

Cataract progression

The presence of exfoliative material in the eye

Peripheral radial iris transillumination defects


Please select an answer
Feedback: Chronic primary angle-closure glaucoma is associated with hyperopia and cataract progression, as well as with
increasing age. Although exfoliation syndrome glaucoma is usually a secondary open-angle glaucoma, there is also a
clinical association of chronic angle-closure glaucoma with exfoliation syndrome. Eyes with exfoliation syndrome may have
peripapillary iris transillumination defects. Peripheral radial iris transillumination defects are found in eyes with pigmentary
dispersion and pigmentary glaucoma in which the anterior chamber angle is typically very deep with slight concavity of the
peripheral iris. There is no clinical association of pigmentary glaucoma with chronic angle-closure glaucoma.

Question 32 of 130
Laser iridotomy is not indicated in which of the following?

Inability to adequately view trabecular meshwork in an eye with narrow angle prior to performing laser trabeculoplasty

Pseudophakic pupillary-block glaucoma

Chronic primary angle-closure glaucoma

Neovascular glaucoma
Please select an answer
Feedback: Laser iridotomy is indicated for phakic, pseudophakic, or aphakic pupillary block and for relative pupillary block
(acute angle-closure and chronic angle-closure glaucoma). If an angle has no peripheral anterior synechiae, but is narrow
enough to prevent performance of a laser trabeculoplasty, an iridotomy is appropriate. In neovascular glaucoma, the iris is
pulled into the trabecular meshwork by fibrovascular proliferation rather than pushed into the angle by relative pupillary
block. The former mechanism would not favorably respond to a laser iridotomy.

Question 33 of 130
Which of the following is not a reason for an increased mean deviation on automated threshold perimetry?

Topical miotic (cholinergic) therapy

Glaucoma progression

High false-positive rate

Cataract progression
Please select an answer
Feedback: Cataract progression, the addition of topical miotic therapy, and glaucoma progression can each cause an
increased mean deviation. A progression of glaucomatous defects may cause an increased mean deviation with or without
an increased pattern standard deviation. A high false-positive rate, which indicates that the patient responded when no
stimulus was presented, would generally not affect or would decrease the mean deviation. However, if a high false-positive
rate is accompanied by supranormal thresholds, the mean deviation may be very small or positive.

Question 34 of 130
Two days after a trabeculectomy, a patient has an intraocular pressure of 3 mm Hg with a large bleb, no leak, and shallow
but formed anterior chamber. On the third day, she presents stating that she developed moderate pain and decreased vision
after bending over. The visual acuity is finger counting and the intraocular pressure is 37 mm Hg. The bleb is unchanged in
appearance. There is a moderate-sized, dark, temporal choroidal detachment. The lens and vitreous are clear, and there is
no evidence of a retinal detachment. Which of the following is not an appropriate action at this time?

Add cycloplegic therapy

Perform drainage of choroidal hemorrhage

Add topical beta blocker to reduce intraocular pressure

Continue topical corticosteroid therapy


Please select an answer
Feedback:

After a delayed, postoperative suprachoroidal hemorrhage of limited to moderate size, it is appropriate to continue topical
corticosteroid therapy and continue, or add cycloplegic therapy. Analgesic therapy for pain is appropriate, and a topical beta
blocker and/or oral carbonic anhydrase inhibitor can be employed to control elevated intraocular pressure. In the case
described, drainage of choroidal hemorrhage would be the least appropriate action.

Question 35 of 130
What would be the most important finding suggestive of glaucoma in a patient with elevated intraocular pressure?

Bilateral cup-to-disc ratio of 0.7

Cup-to-disc ratio asymmetry of 0.1

Very deep optic cup

Cup-to-disc ratio of 0.4 with notch formation in optic nerve rim


Please select an answer
Feedback: Although a cup-to-disc ratio asymmetry of 0.1 to 0.2 may be normal, a difference greater than 0.2 is suggestive
of glaucoma. A bilateral cup-to-disc ratio of 0.7 does not necessarily indicate glaucoma. This can be seen in individuals with
very large optic nerves and accompanying large optic cups. The depth of the optic cup has very little correlation with
glaucoma and glaucomatous optic nerve damage. A cup-to-disc ratio of 0.4 may not arouse much suspicion for glaucoma;
however, if accompanied by elevated intraocular pressure and notch formation in the optic nerve rim, it would be strongly
suggestive of glaucomatous optic nerve damage.
Question 36 of 130
A 66-year-old man had uncontrolled intraocular pressure on glaucoma medical therapy with a previous laser peripheral
iridotomy for chronic angle closure. Five days after a trabeculectomy, he has a very shallow peripheral and central anterior
chamber, patent iridotomy, intraocular pressure of 40 mm Hg, and no evidence of choroidal detachment on ocular B-scan
ultrasonography. Which of the following would not help in the management of this patient?

Vitrectomy

Oral carbonic anhydrase inhibitor therapy

Topical beta blocker therapy

Miotic (cholinergic) therapy


Please select an answer
Feedback: Topical beta blocker therapy and oral carbonic anhydrase inhibitor therapy can reduce intraocular pressure in
eyes with aqueous misdirection (malignant glaucoma). Topical cycloplegic (anticholinergic) therapy can reduce the block in
aqueous misdirection syndrome by tightening the zonules and causing a posterior displacement of the lens. Miotic (cholin-
ergic) therapy tends to exacerbate the block and increase inflammation. Vitrectomy may be necessary if medical
management is not successful.

Question 37 of 130
A 78-year-old man experienced unilateral sudden loss of vision 1 year previously. Currently, he complains of severe pain in
that eye. Examination reveals no light-perception vision, intraocular pressure of 72 mm Hg, iris neovascularizatioll, and
evidence of a central retinal vein occlusion. What would the least helpful topical therapeutic agent be at this time?

Beta blocker

Corticosteroid

Cholinergic (miotic)

Cycloplegic
Please select an answer
Feedback: The management of a painful blind eye with end-stage glaucoma can include topical cycloplegia (anticholinergic
agent) and a corticosteroid agent for comfort. A topical beta blocker may provide relatively little intraocular pressure lowering
but may provide some increased comfort. Eyes with end-stage iris neovascularization and neovascular glaucoma have
completely or near completely closed angles. A cholinergic agent, such as pilocarpine, will not successfully reduce
intraocular pressure in these eyes and will often increase pain and inflammation.
Question 38 of 130
Which problem requires the most urgent management after glaucoma filtering surgery?

Choroidal hemorrhage

Choroidal effusion

Shallow anterior chamber with lens-to-cornea touch

Shallow anterior chamber with iris-to-cornea touch


Please select an answer
Feedback: A shallow anterior chamber with lens-to-cornea touch requires immediate intervention. Rapid corneal
decompensation and cataract progression can occur if this persists. A shallow anterior chamber with iris-to-cornea touch,
but without lens-to-cornea touch is a common transient finding after glaucoma filtering surgery and is usually of no
consequence. Choroidal effusion is a common finding after glaucoma filtering surgery and usually resolves when intraocular
pressure climbs above a hypotonous level. Drainage of choroidal effusion is seldom necessary unless it accompanies lens-
to-cornea touch. A choroidal hemorrhage may require drainage; however, in most cases it can be observed or, if necessary,
drained at a later time.

Question 39 of 130
A 65-year-old man with severe proliferative diabetic retinopathy underwent a very heavy laser photocoagulation treatment
session by your retinal associate 1 day previously. Today, the patient presents with mild pain, blurred vision, and an
intraocular pressure of 45 mm Hg. Your retinal associate has already treated the patient with a topical beta blocker and oral
carbonic anhydrase inhibitor and has referred him to you for further management of elevated intraocular pressure. The
patient has no previous history of glaucoma and no evidence of iris neovascularization. On your examination, the anterior
chamber appears very shallow and the fellow eye has a deep anterior chamber. What would the most appropriate initial
management step be?

Give a topical cycloplegic agent

Perform a laser iridotomy

Perform a laser iridoplasty

Perform a trabeculectomy
Please select an answer
Feedback: Very heavy panretinal laser photocoagulation can cause swelling and anterior rotation of the ciliary body, which
does not respond to a laser iridotomy. The best initial step would be to administer topical cycloplegic therapy. This,
combined with a topical corticosteroid, may cause a posterior rotation and opening of the angle without additional therapy. If
the angle closure fails to respond to medical therapy, then a laser iridoplasty would be the next step. Typically, this is
performed with the argon laser using a low power, long duration, and large spot size. Examples of parameters are 0.2 to 0.5
sec duration, 200 to 300 mW of power, and 200 to 500 nanometer spot size. Topical cycloplegia and then laser iridoplasty
would be indicated before trabeculectomy.
Question 40 of 130

Which statement is incorrect regarding the topical selective beta blocker betaxolol (Betoptic)?

It can be safely used in patients with congestive heart failure.

It is less effective in lowering intraocular pressure than levobunolol (Betagan) or timolol (Timoptic).

It is safer for patients with mild, intermittent asthma attacks.

It has more additive effect of lowering intraocular pressure when combined with dipivefrin (Propine) than do the
nonselective beta blockers.

Please select an answer

Feedback: The nonselective beta blockers levobunolol (Betagan) and timolol (Timoptic) are more effective than betaxolol
(Betoptic) in lowering intraocular pressure. The relative beta-l selectivity of betaxolol allows for safer use in patients with
mild, intermittent asthma. A greater additive effect of dipivefrin (Propine) with betaxolol has been demonstrated over
dipivefrin with nonselective beta blockers. Beta blockers should not be used in patients with congestive heart failure. Both
beta-l selective and nonselective agents can exacerbate heart failure.

Question 41 of 130
Which syndrome includes aniridia with cerebellar ataxia and mental retardation?

Weyers syndrome

Lowe's syndrome

Gillespie syndrome

WAGR syndrome
Please select an answer
Feedback: Aniridia is a bilateral condition in which there is variable iris hypoplasia. Approximately 50-75% of patients with
aniridia develop glaucoma. Gillespie syndrome is an autosomal recessive form of aniridia that is associated with cerebellar
ataxia and mental retardation occurring in 2% of patients with aniridia. WAGR syndrome is an autosomal dominant condition
seen in 13% of aniridia patients that includes Wilms tumor, aniridia, genitourinary abnormalities, and mental retardation.
Lowe's syndrome and Weyers syndrome are other systemic congenital disorders that may be associated with childhood
glaucoma.
Question 42 of 130
According to the Ocular Hypertension Treatment Study (OHTS), which one of the following is associated with an increased
risk of converting from ocular hypertension to primary open-angle glaucoma?

Decreasing age

A history of diabetes mellitis

Reduced central corneal thickness

Smaller cup-disc ratio


Please select an answer
Feedback: The Ocular Hypertension Treatment Study (OHTS) is a multicenter randomized clinical trial designed to evaluate
the safety and efficacy of topical ocular hypotensive medications in preventing or delaying the development of primary open
angle glaucoma in subjects with ocular hypertension. This study also identified baseline demographic and clinical features
that predicted which participants were more likely to progress to primary open angle glaucoma. Enrolled patients were
randomized to observation or treatment with topical glaucoma medications to lower IOP by 20% and maintain IOP less than
or equal to 24 mm Hg. After 5 years of follow-up, 4.4% of treated patients developed glaucoma compared with 9.5% in the
untreated observation group. Higher IOP, reduced central corneal thickness, increased cup-disc ratio, older age, and higher
pattern standard deviation on visual field testing were identified as significant risk factors for the development of primary
open angle glaucoma in patients with ocular hypertension. Diabetes mellitis was a negative risk factor.

Question 43 of 130
Which of the following drugs has been designated by the FDA to be safest during pregnancy?

Brimonidine (Alphagan)

Dorzolamide (Trusopt)

Latanoprost (Xalatan)

Timolol (Timoptic)
Please select an answer
Feedback: Unfortunately, there is little definitive information concerning the use of glaucoma medications during pregnancy.
The FDA has designated brimonidine (Alphagan) as a class B agent, and all other glaucoma agents are class C. Carbonic
anhydrase inhibitors have been shown to be teratogenic in rodents, and prostaglandins increase uterine contractility. In
general, it is prudent to minimize the use of glaucoma medications in pregnant women whenever possible.

Question 44 of 130
Which one of the following is an example of a valved aqueous shunt (glaucoma drainage implant)?

Schocket implant

Molteno implant

Baerveldt implant

Ahmed implant
Please select an answer
Feedback: Aqeous shunts are devices that are frequently used in the surgical management of glaucoma. All modern
aqueous shunts share a common design consisting of a tube that is connected to an end plate. Generally the tube is
inserted into the anterior chamber and shunts aqueous humor to the end plate located in the equatorial region of the globe.
Valved implants incorporate a valve in their design that limits flow through the device when the IOP becomes too low
(usually less than 8-10 mm Hg). The Ahmed implant is the most widely used valved implant. Nonvalved implants allow a free
flow of aqueous humor through the device. The Molteno, Baerveldt, and Schocket implants are all examples of nonvalved
implants. To avoid hypotony in the early postoperative period, temporary restriction of flow is required when using nonvalved
implants until fibrous encapsulation of the end plate occurs.

Question 45 of 130
Which one of the following was a conclusion of the Tube Versus Trabeculectomy (TVT) Study?

Trabeculectomy with MMC had a higher surgical success rate than tube shunt surgery had.

Trabeculectomy with MMC was associated with a higher rate of intraoperative complications than tube shunt surgery
was.

A higher rate of postoperative complications was observed after trabeculectomy with mitomycin C (MMC) compared
with tube shunt surgery.

Tube shunt surgery produced greater IOP reduction than trabeculectomy with MMC produced.
Please select an answer
Feedback: The TVT Study is a multicenter, randomized, clinical trial comparing the safety and efficacy of tube shunt surgery
and trabeculectomy with MMC in patients who had previous cataract extraction with intraocular lens implantation and/or
failed filtering surgery. Enrolled patients were randomized to receive a trabeculectomy with MMC (0.4 mg/ml for 4 minutes)
or 350-Baerveldt glaucoma implant. Trabeculectomy with MMC produced greater IOP reduction during the first 3 months
postoperatively, but similar IOP reduction was observed thereafter. Tube shunt surgery had a higher surgical success rate
than trabeculectomy with MMC. Intraoperative complications occurred at a similar rate with both surgical procedures.
However, postoperative complications were more frequent after trabeculectomy with MMC compared with tube shunt
surgery.

Question 46 of 130
Alpha-2 agonists such as brimonidine (Alphagan) and apraclonidine (Iopidine) should be avoided in which type of patient?

Asthmatics

Pseudophakic

Anemic

Infants
Please select an answer
Feedback: Alpha-2 agonists should be avoided in infants because of an increased risk of respiratory depression,
hypotension, and seizures. These side effects are presumably due to increased CNS penetration of the drug in children.
Beta blockers should be avoided in patients with asthma, but alpha-2 agonists may be used safely. Anemia and
pseudophakia are not contraindications to the use an alpha-2 agonist.
Question 47 of 130
Which one of the following visual field testing strategies may allow an earlier detection of glaucoma compared with standard
automated perimetry?

Suprathreshold testing

Optical coherence tomography (OCT)

Confocal scanning laser ophthalmoscopy (CSLO)

Frequency-doubling technology (FDT)


Please select an answer
Feedback: FDT presents a low spatial frequency grating during visual-field testing that preferentially activates the M cells.
Whether it is because of isolation of specific cell populations that are susceptible to early damage in glaucoma or because of
the reduced redundancy allowing earlier detection of defects, FDP may allow the earlier detection of glaucoma than
standard automated (white-on-white) perimetry. Suprathreshold testing presents a stimulus expected to be brighter than
threshold, and is designed for screening purposes to detect moderate to severe field defects. CSLO and OCT are newer
techniques to provide quantitative measurement of the optic nerve head and retinal nerve fiber layer that may aid clinicians
in making an earlier diagnosis of glaucoma, but they are not visual-field testing strategies.

Question 48 of 130
Which one of the following is the preferred initial surgical procedure for an infant with primary congenital (infantile) glaucoma
and corneal clouding?

Trabeculectomy

Trabeculotomy

Cyclophotocoagulation

Goniotomy
Please select an answer
Feedback: Primary cogenital glaucoma is generally managed surgically, and angle surgery with goniotomy or trabeculotomy
is the preferred initial approach. A goniotomy involves incising the anterior aspect of the trabecular meshwork via an ab
interno approach under gonioscopic guidance. A clear cornea is required to adequately visualize the anterior chamber angle
during goniotomy. In a trabeculotomy, a trabeculotome or prolene suture is inserted into Schlemm's canal from an external
incision and passed into the anterior chamber. Trabeculotomy is a type of angle surgery that can be performed with or
without a clear cornea. Trabeculectomy and cyclodestruction are usually used in the management of primary congenital
glaucoma when angle surgery has failed.
Question 49 of 130
Which ocular condition is associated with an increased risk of complications with cataract surgery?

Pigment dispersion syndrome

Exfoliation syndrome

Ocular hypertension

Angle recession
Please select an answer
Feedback: Patients with exfoliation syndrome have an increased risk of complications with cataract surgery, including lens
dislocation and vitreous loss. Exfoliation syndrome may be associated with zonular weakness, and phacodenesis may be
identified preoperatively in some cases. Additionally, patients with this condition often dilate poorly. Pigment dispersion
syndrome, ocular hypertension, and angle recession have not been reported to increase the risk of cataract extraction.

Question 50 of 130

Which class of glaucoma medications lower IOP by reducing the rate of aqueous humor formation?

Miotics

Prostaglandin analogues

Carbonic anhydrase inhibitors

Hyperosmotics

Please select an answer

Feedback: Carbonic anhydrase inhibitors decrease aqueous humor formation by inhibiting the activity of carbonic
anhydrase in the ciliary epithelium. Miotics reduce IOP by increasing aqueous humor outflow through the trabecular
meshwork, and prostaglandin analogues increase uveoscleral outflow of aqueous. Hyperosmotic agents draw water from
the vitreous cavity by increasing blood osmolality and creating an osmotic gradient, thereby lowering IOP.
Question 51 of 130
Which of the following statements best describes apraclonidine (Iopidine)?

It is an alpha-1 adrenergic agonist.

It may cause transient lid retraction.

It commonly causes systemic hypotension.

It is associated with macular edema in aphakic eyes.


Please select an answer
Feedback: Apraclonidine (lopidine) can cause conjunctival blanching and lid retraction. Apraclonidine is an alpha-2
adrenergic agonist and, unlike clonidine, does not cause systemic hypotension. Epinephrine and possibly dipivefrin
(Propine) are associated with macular edema in aphakic eyes.

Question 52 of 130
A 21-year-old woman with juvenile open-angle glaucoma and 7 diopters of myopia complains of severe blurring of vision
after using 1 drop of pilocarpine. What is the most likely cause of her symptom?

A small pupil

Increased myopia

Retinal detachment

Increased hyperopia
Please select an answer
Feedback: Young, highly myopic patients may have substantially increased myopia with miotic therapy. This occurs
because of a miotic-induced increased convexity of the lens and forward lens movement. All patients with a normal iris
develop a small pupil on miotic therapy. This can cause nyctalopia and is more troublesome in older patients with a cataract
or other media opacity. Retinal detachment after miotic therapy can occur but would not be the most likely cause of severe
visual blurring in this case.

Question 53 of 130
Two years after a successful filtering procedure (full-thickness sclerectomy), a patient complains of pain, tearing, and blurred
vision for 2 days. The visual acuity is 20/50, the intraocular pressure is 4 mm Hg, the bleb is flat, and there is a rare cell in
the anterior chamber. What is the most likely explanation of these symptoms and signs?

Ciliary body detachment

Endophthalmitis

Bleb leak

Retinal detachment
Please select an answer
Feedback: The patient with a previously high, thin, ischemic bleb is more prone to develop a late bleb leak. This is usually
manifested as mild discomfort, tearing, and blurred vision and may be more likely to occur after full-thickness filtering
procedures or trabeculectomy with antifibrotic therapy. Objective signs include a flat bleb (usually with demonstrable leak),
mildly decreased visual acuity, low intraocular pressure, and minimal or no anterior chamber inflammation. If severe anterior
chamber reaction or hypopyon is seen, endophthalmitis must be suspected. Ciliary body detachment may be seen and is
secondary to hypotony and inflammation in an eye with endophthalmitis, retinal detachment, or bleb leak. A retinal
detachment could explain many of these findings but would be a less likely cause of this clinical picture.

Question 54 of 130
Glaucoma-like visual field defects can not be seen in which condition?

Ischernic optic neuropathy

Retinal vascular occlusion

Cerebrovascular accident

Buried optic nerve drusen


Please select an answer
Feedback: Buried optic nerve drusen, retinal vascular occlusion, and ischemic optic neuropathy all produce optic nerve-type
visual field defects that can mimic glaucomatous visual field loss. A cerebrovascular accident would be expected to produce
a postchiasmal lesion with a homonymous hemianopic or quadrantic defect.

Question 55 of 130
A patient with elevated intraocular pressure undergoes automated static threshold perimetry. Most threshold determinations
are high (40 dB to 50 dB). What is the most likely reason for this?

Drowsy patient

End-stage glaucoma

Alert but nervous patient

Media opacity
Please select an answer
Feedback: A drowsy patient would be expected to have a high false-negative rate (the patient fails to respond to a
previously seen stimulus) and possibly also abnormally low thresholds either diffusely throughout the visual field or in an
irregular pattern. Media opacity would also tend to diffusely decrease thresholds. End-stage glaucoma can produce a
substantial decrease in some or all thresholded spots. An alert but nervous patient may have high thresholds accompanied
by a high false-positive rate (the patient responds when no stimulus is presented).

Question 56 of 130
A miotic agent would be least effective in a patient with glaucoma and which one of the following?

Aphakia

Aniridia with open angle

Angle recession

Severe secondary angle closure


Please select an answer
Feedback: In the absence of substantial secondary angle closure, aniridia does not reduce the effectiveness of topical
miotic (cholinergic) therapy. The effect of miotic agents is mediated through the ciliary muscle and not the pupillary
sphincter, which is absent in patients with aniridia. Surgical aphakia does not alter the effectiveness of miotic therapy. Angle
trauma and angle recession can decrease the effectiveness of miotic therapy. Eyes with severe synechial angle closure
would be the least likely to respond to cholinergic agents and may have a paradoxical rise of intraocular pressure from
miotic therapy because of a reduction of nonconventional uveoscleral outflow.

Question 57 of 130
Which of the following statements best describes corticosteroid-induced intraocular pressure elevation?

It is more common in patients with primary open-angle glaucoma than in patients with ocular hypertension.

Fluorinated corticosteroids usually cause a greater incidence of intraocular pressure elevation than nonfluorinated
corticosteroid preparations.

It usually begins within 1 day after beginning corticosteroid therapy.

Intraocular pressure usually does not return to baseline levels after discontinuing the corticosteroid.
Please select an answer
Feedback: Corticosteroid-induced intraocular pressure elevation usually begins about 2 to 4 weeks after initiation of
corticosteroid therapy. Intraocular pressure often returns to baseline levels after discontinuation of the corticosteroid.
Fluorinated corticosteroids (eg, fluorometholone) are less likely to cause intraocular pressure elevation than nonfluorinated
corticosteroids. Corticosteroid responsiveness is more likely in patients with primary open-angle glaucoma than in patients
with ocular hypertension or patients without intraocular pressure elevation.

Question 58 of 130
Which of the following is the most helpful clue in the diagnosis of chronic primary angle-closure glaucoma?

Gonioscopic findings

Ocular symptoms (pain, haloes)

Aamount of glaucomatous optic nerve damage at presentation

Level of intraocular pressure at presentation


Please select an answer
Feedback: Patients with chronic primary angle-closure glaucoma can present with intraocular pressure that is low, normal,
or elevated. Ocular symptoms may or may not be present. There may be any degree of glaucomatous optic nerve damage
or no damage at all. Gonioscopic findings, preferably with the Zeiss or Posner lens, are the key to the diagnosis of chronic
primary angle-closure glaucoma.
Question 59 of 130
Lens extraction will not resolve glaucoma in which of the following situations?

Microspherophakia

Exfoliation syndrome (pseudoexfoliation) glaucoma

Phacolytic glaucoma

Chronic primary angle-closure glaucoma


Please select an answer
Feedback: Lens extraction might resolve glaucoma in microspherophakia, phacolytic glaucoma, and chronic primary angle-
closure glaucoma. Exfoliation syndrome glaucoma would not be substantially improved by cataract surgery. The material is
produced by nonpigmented ciliary epithelium and other ocular tissues and can be found in pseudophakic and aphakic eyes
on the capsule, vitreous, corneal endothelium, iris, and anterior chamber angle.

Question 60 of 130

Which statement does not describe chronic primary angle-closure glaucoma?

It can develop in a myopic eye.

It often causes no pain.

It can develop in a patient with primary open-angle glaucoma.

It can be prevented by pilocarpine therapy.

Please select an answer

Feedback: Chronic primary angle-closure glaucoma more commonly develops in hyperopic eyes with shorter axial length
and crowded eripheral anterior chamber. However, angle-closure glaucoma can develop in the myopic eye, especially one
with an enlarging, progressive nuclear sclerotic cataract. Chronic angle-closure glaucoma can develop in an eye with
previous primary open-angle mechanism (combined-mechanism glaucoma). Pain is uncommon in chronic angle-closure
glaucoma even late in the course of the disease, when substantial intraocular pressure elevation can occur. Pilocarpine
therapy usually does not relieve pupillary block. Pupillary block can be increased with miotic therapy, and further angle
closure can occur.
Question 61 of 130
Miotic therapy would help and not worsen the control of intraocular pressure in which one of the following?

Neovascular glaucoma

Exfoliation syndrome glaucoma

Ciliary-block (aqueous misdirection) glaucoma

Microspherophakia
Please select an answer
Feedback: Miotic (cholinergic) therapy includes pilocarpine, carbachol, bromide, and echothiophate iodide. These agents
reduce intraocular pressure by increasing trabecular outflow. Miotic therapy can worsen ocular pressure control in ciliary-
block glaucoma because ciliary body contraction loosens the lens zonules, which causes the lens to move farther forward,
increasing the ciliary block. In microspherophakia, the pupillary block induced by the abnormally rounded lens can also be
worsened with miotic therapy. In neovascular glaucoma, there may be substantial angle closure and little beneficial effect
from miotic therapy. A cycloplegic (anticholinergic) agent, such as atropine, would be preferred in a patient with neovascular
glaucoma. Exfoliation syndrome glaucoma, however, should respond well to miotic (cholinergic) therapy.

Question 62 of 130
Which one of the following does not have a hereditary basis?

Cup/disc ratio

Corticosteroid-induced pressure responsiveness

Formation of an encapsulated bleb (Tenon's cyst) after trabeculectomy

Primary open-angle glaucoma


Please select an answer
Feedback: Primary open-angle glaucoma is a hereditary disorder. People with this disorder are likely to demonstrate
corticosteroid-induced intraocular pressure elevation. The cup/disc ratio also, in part, appears to be affected by heredity.
People with a large cup/disc ratio may be more likely to sustain glaucoma damage with elevated intraocular pressure. The
development of an encapsulated bleb (Tenon's cyst) after trabeculectomy may be associated with previous laser
trabeculoplasty and previous sympathomimetic or beta-adrenergic antagonist use. No hereditary basis for the development
of an encapsulated bleb has been demonstrated.

Question 63 of 130
Which statement does not accurately describe argon laser trabeculoplasty?

It is effective in pigmentary glaucoma.

The effect of an argon laser trabeculoplasty tends to diminish over time.

It is less effective in patients with previous blunt ocular trauma.

A repeat argon laser trabeculoplasty is about as effective as the initial procedure.


Please select an answer
Feedback: Pigmentary glaucoma, exfoliation syndrome glaucoma, and primary open-angle glaucoma tend to have a
reasonably good pressure-lowering response to argon laser trabeculoplasty. The procedure is less effective in aphakic and
pseudophakic eyes and eyes with glaucoma after blunt ocular trauma. The response of argon laser trabeculoplasty tends to
wear off over time, and a repeat procedure tends to be less effective.

Question 64 of 130
Topical ocular beta blockers would not have a beneficial effect on which one of the following disorders?

Systemic hypertension

Second-degree heart block

Supraventricular tachyarrhythmia

Angina pectoris
Please select an answer
Feedback: Oral beta blocker therapy has been used for the control of supraventricular tachycardia, and for the treatment of
systemic hypertension and angina pectoris. Substantial systemic levels of beta blockers can occur with topical ocular beta
blocker therapy. Topical ocular beta blocker therapy can exacerbate second-degree heart block and should be avoided in
these patients.

Question 65 of 130
Which statement does not accurately describe dipivefrin (Propine)?

It is more lipophilic than topical ocular epinephrine formulations.

It is more likely to cause contact dermatitis than a topical ocular beta adrenergic antagonist.

Systemic effects are equally likely with dipivefrin and epinephrine.

It is formulated in a lower concentration than the epinephrine formulations.


Please select an answer
Feedback: Dipivefrin (Propine) is more lipophilic than topical ocular epinephrine and therefore penetrates the cornea better.
This allows its formulation at a 0.1% strength rather than the typical formulations of epinephrine (0.5%, 1%, and 2%).
Dipivefrin is a prodrug that is transformed by corneal esterases into the active agent epinephrine. Contact dermatitis is a
common complication of chronic dipivefrin therapy. Dipivefrin has been shown to cause fewer cardiovascular effects than
epinephrine.

Question 66 of 130
Topical ocular beta blockers have not been reported to cause which side effect listed below?

Exacerbation of myasthenia gravis

Hypokalemia

Blockage of the systemic response to hypoglycemia in diabetic patients

Heart block
Please select an answer
Feedback: Topical ocular beta blockers have been reported to cause heart block, exacerbation of myasthenia gravis, and
blockage of the systemic response to hypoglycemia in diabetic patients. Hypokalemia is more likely to occur with oral
carbonic anhydrase inhibitor therapy, especially with concurrent use of a potassium-depleting diuretic such as furosemide,
hydrochlorothiazide, or chlorthalidone.

Question 67 of 130
A pars plana vitrectomy would not help the treatment of glaucoma in which clinical situation?

Glaucoma with a chronic vitreous hemorrhage in an aphakic eye

Ciliary-block (aqueous misdirection) glaucoma

An eye with active uveitis and vitritis

After cataract surgery when a broken capsule leads to retained cortex and nuclear debris
Please select an answer
Feedback: A pars plana vitrectomy can be beneficial in patients with glaucoma and substantial amounts of retained cortex
and other lens debris after cataract surgery in which the capsule was ruptured. Lens debris can directly obstruct the
trabecular meshwork or incite inflammation, which can also cause intraocular pressure elevation. If there is a chronic
vitreous hemorrhage, particularly in an aphakic or pseudophakic eye, a ghost-cell glaucoma mechanism may develop.
Vitrectomy map help in this situation as well. Vitrectomy may break the blockage that occurs in ciliary-block (aqueous
misdirection) glaucoma. To be successful, however, there should be a disruption of the anterior vitreous face. In an eye with
active intraocular inflammatory disease, topical or systemic cortico-steroid therapy may help reduce inflammation and
secondarily improve intraocular pressure control. Periocular injections may cause steroid-induced glaucoma that is not
easily reversed, though such an injection may be considered if it has been established that the patient is NOT a steroid
responder. Typically, a vitrectomy is not recommended to improve intraocular pressure control in an eye with vitritis.
However, there may be other clinical indications to perform a vitrectomy in any eye with vitiritis, such as severe vitritis,
suspecited P. acnes or other infection, or ocular lymphoma.

Question 68 of 130
Problems with apraclonidine (Iopidine) do not include which one of the following?

Systemic hypotension

Red eye

Allergy

Tachyphylaxis
Please select an answer
Feedback: Apraclonidine (Iopidine), an agonist, reduces aqueous humor production. A chronic red eye may be seen with
this medication, as is also seen with epinephrine and dipivefrin (Propine). An allergy to medication is not uncommon.
Although apraclonidine initially has the notable short-term effect of decreasing pressure, tachyphylaxis may develop, leading
to a decreased responsiveness to the medication after several weeks or months. The oral antihypertensive clonidine can
cause substantial systemic hypotension. Topical apraclonidine does not cause this problem.
Question 69 of 130
A 58-year-old man presents to your office with a history of primary open-angle glaucoma and intraocular pressures of 20
mm Hg OU using a topical beta blocker twice daily and pilocarpine 4%, 3 times daily to both eyes. Gonioscopy reveals open
angles and light trabecular pigmentation. You dilate the patient's pupils with two sets of tropicamide 1% and phenylephrine
2.5% drops in each eye. One hour later, you return to perform the dilated examination and the patient complains of blurred
vision. There is mild corneal edema, and the intraocular pressure is 44 mm Hg bilaterally. Which of the following is the most
likely reason for this acute elevation of intraocular pressure?

Hypersensitivity to one of the dilating agents

Idiosyncratic reaction to one of the dilating agents

Angle closure

Reversal of intraocular pressure-lowering effect of glaucoma medication by one of the dilating agents
Please select an answer
Feedback: After dilation of a patient with primary open-angle glaucoma on a topical beta blocker and topical miotic agent,
there can be a substantial intraocular pressure elevation, in part because of reversal of the cholinergic effect of the miotic
agent. Pigment release may also contribute to intraocular pressure elevation. Hypersensitivity, or an idiosyncratic reaction is
unlikely. Angle closure can occur after dilation, but is a less common cause of elevated intraocular pressure in this clinical
situation.

Question 70 of 130

The topical ophthalmic prostaglandin latanoprost (Xalatan) increases nontrabecular uveoscleral outflow. Which one of the
following results would be expected?

This medication would have no intraocular pressure-lowering effect in an eye with severe chronic angle closure

Latanoprost's effect on intraocular pressure would be pressure dependent

Latanoprost may successfully lower the pressure in an eye with elevated episcleral venous pressure

Little additional intraocular pressure lowering would occur in an eye already receiving topical ocular beta-adrenergic
antagonist therapy

Please select an answer

Feedback: Topical ophthalmic prostaglandin therapy increases uveoscleral outflow. Unlike trabecular outflow, nontrabecular
uveoscleral outflow is not pressure dependent. Medications that increase trabecular outflow, such as pilocarpine, are not
expected to have much beneficial effect on intraocular pressure in an eye with extensive angle closure; however, topical
ocular prostaglandin therapy would be expected to be beneficial in such an eye. Topical beta-adrenergic antagonist therapy
reduces aqueous production, and if a prostaglandin agent were coupled with this, each agent would be expected to
contribute to a lowering of intraocular pressure. Topical ocular prostaglandin therapy would be expected to further lower the
intraocular pressure in an eye with elevated episcleral venous pressure. This therapy may limit trabecular outflow but should
not affect nontrabecular outflow.
Question 71 of 130
Which of the following is a risk factor for developing primary angle-closure glaucoma (PACG)?

Myopia

Male gender

Short axial length

Young age
Please select an answer
Feedback: Patients who develop PACG have small, crowded anterior segments and short axial lengths. The most important
factors predisposing to PACG are a shallow anterior chamber, a thick lens, increased anterior curvature of the lens, a short
axial length, and a small corneal diameter and radius of curvature. PACG occurs more commonly in hyperopic patients.
PACG has been reported to be 2-4 times more common in women than men. The prevalence of PACG rises with increasing
age.

Question 72 of 130
Which variant of iridocorneal endothelial (ICE) syndrome predominantly has corneal changes with minimal iris
abnormalities?

Chandler's syndrome

Axenfeld-Rieger syndrome

Progressive iris atrophy

Cogan-Reese syndrome
Please select an answer
Feedback: Iridocorneal endothelial (ICE) syndrome is a group of disorders characterized by abnormal corneal endothelium,
iris atrophy, and secondary angle-closure glaucoma. The disease is unilateral, more common in women, and usually
presents between 20 and 50 years of age. Progressive iris atrophy, Chandler's syndrome, and Cogan-Reese syndrome are
the 3 clinical variants that exist along a spectrum. In Chandler's syndrome, minimal iris changes are present and corneal
changes predominate. Iris changes predominate in progressive iris atrophy and include progressive atrophy of the iris
resulting in hole formation, corectopia, and ectropian uveae. Iris atrophy tends to be less severe in Cogan-Reese syndrome,
and this condition typically has pedunculated nodules or diffuse pigmented lesions on the anterior iris surface. Axenfeld-
Rieger syndrome is a congenital anomaly that is associated with iris changes that resemble ICE syndrome, but the condition
is bilateral.

Question 73 of 130
Which glaucoma is caused by the leakage of lens proteins through the capsule of a mature or hypermature cataract?

Ectopia lentis

Phacolytic glaucoma

Lens particle glaucoma

Phacomorphic glaucoma
Please select an answer
Feedback: The protein composition of the lens changes with aging, and increased concentration of high-molecular-weight
lens proteins develop over time. In a mature or hypermature lens, these proteins can be released through microscopic
openings in the lens capsule. A secondary open-angle glaucoma may develop as lens proteins, phagocytizing
macrophages, and other inflammatory debris obstruct the trabecular meshwork. Although medications should be used to
treat the IOP elevation, definitive therapy requires cataract extraction. Lens particle glaucoma occurs when lens cortex
particles obstruct the trabecular meshwork following disruption of the lens capsule with cataract extraction or ocular trauma.
In phacomorphic glaucoma, a large, intumescent lens induces angle-closure glaucoma. Ectopia lentis refers to a
displacement of the lens from its normal anatomic position.

Question 74 of 130
Which of the following increases the risk of aqueous misdirection (malignant glaucoma)?

High myopia

Chronic angle-closure glaucoma

Pseudophakia

Prior pars plana vitrectomy


Please select an answer
Feedback: Aqueous misdirection is a rare form of glaucoma that typically presents following ocular surgery. The condition
results from a misdirection of aqueous humor posteriorly. Increased fluid volume in the vitreous cavity pushes the lens-iris
diaphragm forward causing secondary angle-closure glaucoma. Axial shallowing of the anterior chamber is present in this
condition. Patients with a history of angle-closure glaucoma (acute and chronic) are at increased risk of aqueous
misdirection. Prior pars plana vitrectomy does not increase the risk of this complication, and vitrectomy may actually be used
to treat aqeuous misdirection that is refractory to medical and laser therapy. High myopia and pseudophakia are not risk
factors for aqueous misdirection.

Question 75 of 130
Which syndrome involves secondary glaucoma associated with a rhegmatogenous retinal detachment?

Schwartz syndrome

Zellweger syndrome

Lowe's syndrome

Hallermann-Streiff syndrome
Please select an answer
Feedback: Rhegmatogenous retinal detachments are typically associated with low IOP. However, Schwartz first described
IOP elevation associated with a rhegmatogenous retinal detachment. The presumed mechanism of IOP elevation in
Schwartz syndrome involves migration of photoreceptor outer segments through the retinal tear and into the anterior
chamber, where they obstruct aqueous outflow through the trabecular meshwork. Hallermann-Streiff syndrome, Zellweger
syndrome, and Lowe's syndrome are all systemic congenital disorders associated with childhood glaucomas.
Question 76 of 130
Which of the following is a feature of selective laser trabeculoplasty (SLT) compared with argon laser trabeculoplasty (ALT)?

Greater degree of IOP reduction

Less coagulative damage to the trabecular meshwork

Greater proportion of patients respond to treatment

Longer clinical experience with the procedure


Please select an answer
Feedback: Laser trabeculoplasty is a procedure in which laser energy is delivered to the trabecular meshwork to improve
the outflow of aqueous humor. ALT has been in popular use for several decades. SLT is a newer procedure that uses a
frequency-doubled Nd:YAG laser. Several studies have demonstrated that the response rate and degree of IOP reduction
observed after ALT and SLT are similar.

Question 77 of 130
A 50-year-old man has recurrent episodes of markedly elevated IOP and low-grade anterior chamber inflammation. Which is
the most likely diagnosis?

Glaucomatocyclitic crisis (Posner-Schlossman syndrome)

Exfoliation syndrome

Glaucoma associated with elevated episcleral venous pressure

Angle-recession glaucoma
Please select an answer
Feedback: Glaucomatocyclitic crisis (Posner-Schlossman syndrome) is a rare type of open-angle glaucoma typically
affecting middle-aged adults. It usually presents with unilateral eye pain and blurred vision associated with markedly
increased IOP (40-60 mm Hg). A mild iritis is present and resolves spontaneously within a few weeks. The IOP usually
remains normal between episodes, but a chronic secondary glaucoma may develop with an increasing number of attacks.
Angle-recession glaucoma, glaucoma associated with elevated episcleral venous pressure, and exfoliation glaucoma are all
chronic forms of glaucoma that are not typically associated with ocular inflammation.

Question 78 of 130
Which of the following is a characteristic sign of pigment dispersion syndrome?

Blood in Schlemm's canal

Iris bombe

Spoke-like radial iris transillumination defects

Peripupillary atrophy
Please select an answer
Feedback: Mechanical contact between the zonular fibers and iris pigment epithelium causes iris pigment release in
pigment dispersion syndrome. Spoke-like iris transillumination defects in the iris midperiphery develop from the iridozonular
friction and are characteristic of this syndrome. Liberated pigment deposits on the corneal endothelium in a vertical spindle
pattern (Krukenberg spindle), in the trabecular meshwork, and on the lens periphery (Scheie stripe). Peripupillary atrophy is
commonly observed in exfoliation syndrome. Blood in Schlemm's canal may be seen with elevated episcleral venous
pressure. Iris bombe develops in the presence of pupillary block.

Question 79 of 130
Which medication has been reported to cause secondary angle-closure glaucoma in rare cases?

Metoprolol (Toprol)

Prednisone

Topiramate (Topamax)

Azithromycin (Zithromax)
Please select an answer
Feedback: Topiramate (Topamax) is a sulfa medication that has been reported to induce angle-closure glaucoma in rare
instances. The underlying mechanism involves ciliary body congestion and development of a ciliochoroidal effusion that
causes anterior rotation of the ciilary body and angle closure. Systemic corticosteroid therapy can also cause secondary
glaucoma, but this occurs via an open-angle mechanism. Azithromycin and metoprolol have not been reported to cause
secondary angle-closure glaucoma.

Question 80 of 130

A 60-year-old woman with proliferative diabetic retinopathy with a vitreous hemorrhage in the right eye has been followed for
2 months. She presents with severe ocular pain, and her IOP is 40 mm Hg in the right eye. Khaki-colored cells are seen in
the the anterior chamber, and the angle appears open on gonioscopy. What is the most likely diagnosis?

Ghost-cell glaucoma

Neovascular glaucoma

Posner-Schlossman syndrome

Angle-recession glaucoma

Please select an answer

Feedback: Ghost-cell glaucoma is a secondary open-angle glaucoma caused by degenerated red blood cells (ghost cells)
that are less pliable than normal red blood cells and block the trabecular meshwork. Ghost cells have lost their intracellular
hemoglobin and appear as small, khaki-colored cells. These cells generally develop within 1-3 months of a vitreous
hemorrhage. Patients with proliferative diabetic retinopathy are at increased risk to develop neovascular glaucoma, but the
angle is usually closed with this type of glaucoma. Angle-recession glaucoma and primary open angle glaucoma are chronic
forms of glaucoma that are generally not associated with acute IOP rise and do not have khaki-colored cells in the anterior
chamber.
Question 81 of 130
How does laser trabeculoplasty achieve its desired effect of lowering intraocular pressure?

Increasing aqueous outflow

Decreasing episcleral venous pressure

Decreasing aqueous production

Producing microperforations in the trabecular meshwork


Please select an answer
Feedback:

Several studies have demonstrated that laser trabeculoplasty reduces intraocular pressure by improving the facility or
rate of aqueous outflow. Although the mechanism of enhanced outflow is uncertain, many authors have hypothesized
that this may occur from altered structures of the collagen in the trabecular lamellar beams, or release of chemical
mediators that induce a structure altering inflammatory cascade.

Question 82 of 130
A 2-year-old boy has unilateral cutaneous facial redness, ipsilateral glaucoma, and hemiplegia. Which of the following is the
most likely diagnosis?

Sturge-Weber syndrome

Nevus of Ota

Neurofibromatosis

Tuberous sclerosis
Please select an answer
Feedback:

Sturge-Weber syndrome (encephalofacial angiomatosis) is a nongenetically transmitted phacomatosis consisting of


facial cutaneous angioma and ipsilateral leptomeningeal vascular malformation. The neurologic sequelae can include
intracranial calcification, seizures, focal neurologic deficits (hemianopia, hemiparesis) and mental
retardation. Ipsilateral glaucoma can develop anytime between birth and later in childhood. Multiple mechanisms for
the glaucoma have been proposed and include developmental angle abnormalities, elevated episcleral venous
pressure, and hypersecretion of aqueous. Tuberous sclerosis is not associated with glaucoma, whereas the
cutaneous lesions in neurofibromatosis are melanocytic (cafe-au-lait spots) rather than angiomatous. Nevus of Ota is
not associated with cutaneous angioma, hemiplegia, or glaucoma.
Question 83 of 130
Within the Collaborative Normal Tension Glaucoma Study (CNTGS), what feature was associated with an increased risk of
glaucomatous progression?

Asian race

Male gender

History of cardiovascular disease

Disc hemorrhage
Please select an answer
Feedback:

The Collaborative Normal Tension Glaucoma Study was a multicenter randomized clinical trial designed to determine
whether reduction of intraocular pressure ameliorates normal tension glaucoma. Eligible patients were randomly
assigned to close observation or treatment to lower intraocular pressure by 30% from baseline. To be randomized,
eyes met criteria for the diagnosis of normal tension glaucoma and had either documented progression, high-risk
visual field defects that threatened fixation, or development of a new disc hemorrhage. Disc hemorrhages, migraines,
and non-Asian race were identified as significant risk factors for visual field progression in normal tension glaucoma
in multivariate analysis.

Question 84 of 130
What commonly causes failure to control the intraocular pressure after glaucoma filtering surgery?

Endophthalmitis

Retinal detachment

Episcleral fibrous

Overfiltration
Please select an answer
Feedback:

A common cause of failure to control intraocular pressure after glaucoma filtering surgery is episcleral fibrosis. To
reduce the degree of postoperative fibrosis, antifibrotic agents are routinely used as adjuncts to trabeculectomy.
Mitomycin C and 5-fluorouracil are antifibrotic agents in common use. When used intraoperatively during glaucoma
surgery, these agents are applied between the sclera and conjunctiva.
Question 85 of 130
Which of the following is considered to be the mechanism of glaucoma in aniridia?

Elevated episcleral venous pressure

Increased central corneal thickness

Angle-closure glaucoma

Lenticular abnormalities
Please select an answer
Feedback:

At birth, the aniridic iridocorneal angle appears open. Congenital glaucoma is rare in infants with aniridia. However,
over the first two decades of life, the position of the rudimentary iris leaflet gradually appears more anterior, covering
trabecular meshwork. This results in secondary chronic angle closure glaucoma. Although increased central corneal
thickness has been reported in aniridia, this is not a mechanism of glaucoma, but may obscure a diagnosis.
Glaucoma secondary to lenticular intumescence or ectopic lentis has also been reported, although very rare.
Elevated episcleral venous pressure is not associated with aniridia.

Question 86 of 130
In the Advanced Glaucoma Intervention Study (AGIS), what factor was identified for trabeculectomy failure?

Older age

Hypertension

Lower intraocular pressure (IOP)

Diabetes
Please select an answer
Feedback:

The AGIS was a randomized clinical trial which compared the order of application of two treatment strategies. This
order of intervention was either TAT (trabeculectomy - first line, argon laser trabeculoplasty - second line,
trabeculectomy - third line therapy) or ATT (argon laser trabeculoplasty - first line, trabeculectomy - second line,
trabeculectomy - third line therapy). The second and third treatment interventions were offered only after failure of the
previous intervention, based on number of medications, IOP elevation, visual field defect, and optic disc damage.

AGIS 11 identified pre-intervention and post-intervention risk factors for the failure of both argon laser trabeculoplasty
and trabeculectomy. Highly significant (P<0.01) risk factors for failure of trabeculectomy included younger age, high
baseline intraocular pressure, diabetes, and any post-operative complication. The post-operative complications most
highly associated with failure were marked post-operative inflammation and post-operative elevation of intraocular
pressure. Prior trabeculectomy did not significantly affect the failure rate of subsequent interventions. The study's
authors noted that most AGIS trabeculectomies were performed between 1988 and 1993, and may not reflect current
surgical technique.
Question 87 of 130
What is the most common extra-ocular abnormality in Rieger syndrome?

Congenital heart defect

Deafness

Redundant periauricular skin

Smaller than normal teeth


Please select an answer
Feedback:

Rieger syndrome is part of the spectrum of bilateral, hereditary, anterior-segment dysgenesis resulting from the
developmental arrest of neural crest cells (neuroectoderm). Axenfelds anomaly is characterized by posterior
embryotoxon (prominent anteriorly displaced Schwalbes line) with prominent iris processes. Riegers anomaly is
Axenfelds anomaly in addition to iris abnormalities including stromal hypoplasia, ectropion uveae, corectopia, or iris
holes. Riegers syndrome is Riegers anomaly with a developmentally associated extra-ocular abnormality such as
microdontia (small teeth), hypodontia (fewer teeth), and craniofacial abnormalities are most commonly observed,
because teeth and facial bones are derived from neural crest cells. Less common extra-ocular findings include
umbilical anomalies, cardiac malformations, and sensorineural hearing loss.

Question 88 of 130
For patients undergoing trabeculectomy, what risk factor is associated with the development of suprachoroidal hemorrhage?

High preoperative intraocular pressure (IOP)

Hyperopia

Young age

Phakia
Please select an answer
Feedback: There are numerous systemic and ocular risk factors for suprachoroidal hemorrhage. Systemic risk factors
include advanced age, systemic vascular disease, hypertension, vascular disease, and clotting disorders. Ocular risk factors
include hypotony (especially after marked elevated preoperative intraocular pressure), choroidal effuision, history of prior
suprachoroidal hemorrhage in the fellow eye, high myopia, previous vitrectomy, and intraocular vascular anomalies.
Question 89 of 130

Which of the following general anesthetic agents can lower the intraocular pressure in a child?

Midazolam

Halothane

Succinylcholine

Ketamine
Please select an answer
Feedback:

Inhalational halothane anesthetics results in rapid lowering of intraocular pressure. Some proposed mechanisms
include contraction of choroidal volume and changes in cardiovascular tone. In prospective, randomized trials, neither
ketamine or midazolam had pressure-lowering effects. Succinylcholine induction can elevate intraocular pressure by
causing co-contraction of antagonistic extraocular muscles.

Question 90 of 130

Mutation in the TIGR/myocilin gene is associated with which of the following disorders?

Juvenile open-angle glaucoma

Pigment dispersion syndrome

Pseudoexfoliation

Nanophthalmos

Please select an answer

Feedback:

The gene GLC1A, was the first open-angle glaucoma gene identified. It was initially mapped in a large juvenile
glaucoma family. The gene has been localized to chromosome 1. Mutations in this gene, which produce the protein
myocilin, was concurrently found to be upregulated in trabecular meshwork cells following dexamethasone exposure,
for which it was functionally termed trabecular meshwork induced glucocorticoid response (TIGR). Mutations in the
TIGR/myocilin gene are not limited to juvenile glaucoma and have been reported in 3% of individuals with adult-onset
primary open-angle glaucoma.
Question 91 of 130
In the Advanced Glaucoma Intervention Study (AGIS), which of the following risk factors was most closely associated with
failure of laser trabeculoplasty?

High preoperative intraocular pressure (IOP)

Advanced age

Hypertension

Previous trabeculectomy
Please select an answer
Feedback:

The Advanced Glaucoma Intervention Study is a multicenter randomized clinical trial that compared a sequence of
treatment beginning with argon laser trabeculoplasty or trabeculectomy for managing patients with medically
uncontrolled glaucoma. AGIS identified preintervention and postintervention risk factors for the failure of both argon
laser trabeculoplasty and trabeculectomy. Younger age and high baseline IOP were identified as highly significant (P
< 0.01) risk factors for failure of trabeculoplasty. Diabetes and any postoperative complication were identified as
borderline significant (0.01 < P 0.05) risk factors for trabeculoplasty failure. Prior trabeculoplasty or trabeculectomy
did not significantly affect the failure rate of subsequent interventions.

Question 92 of 130
What is the stimulus used in short-wavelength automated perimetry (SWAP)?

Size V, blue

Size III, yellow

Size V, yellow

Size III, blue


Please select an answer
Feedback:

Glaucoma has been associated with short-wavelength (blue) color vision deficits. Because sensitivity to blue stimuli is
believed to be mediated by a small subpopulation of morphologically distinct ganglion cells, the small bistratified
ganglion cells, they appear to be ideal targets to detect early glaucomatous loss. A size-V blue stimulus and yellow
background are the stimuli typically used when referring to short wavelength automated perimetry (SWAP). Smaller
stimuli (Size III, blue) has lower detection. Since the number of blue cones is small, the perimetric variability is higher
for a smaller stimuli. Yellow is the complementary blue but does not correspond to a color receptive field shown to be
selectively reduced in glaucoma.
Question 93 of 130
Which of the following disorders may cause elevated episcleral venous pressure (EVP)?

Aniridia

Central retinal vein occlusion

Pigment dispersion syndrome

Thyroid eye disease


Please select an answer
Feedback:

Episcleral venous pressure (EVP) is normally 8-10 mm Hg. EVP may become elevated by a variety of clinical entities
that either obstruct venous outflow or involve arteriovenous malformations. Thyroid eye disease and retrobulbar
tumors are conditions that may produce venous obstruction and secondary elevation of EVP. Carotid-cavernous
sinus fistulas and Sturge-Weber syndrome are examples of arteriovenous malformations that can cause elevated
EVP. Because aqueous humor ultimately drains into the episcleral venous system, elevation of EVP can produce an
increase in intraocular pressure and glaucoma. These conditions may cause a central retinal vein occlusion but not
be caused by it. Pigment dispersion syndrome and aniridia cause glaucoma, but not through the mechanism of
elevated episcleral venous pressure.

Question 94 of 130
Which of the following medications, which is used as an adjunct to trabeculectomy, must be activated by enzymatic
cleavage prior to alkylating DNA and prohibiting cell division?

5-fluorouracil

Vincristine

Mitomycin C

Vinblastine
Please select an answer
Feedback:

Mitomycin C (MMC) is a compound derived from Streptomyces caespitosus and serves as an antibiotic as well as an
antineoplastic agent. MMC is a potent inhibitor of fibroblasts and alters the amount of scarring postoperatively. the
inhibition of MMC is activated by enzymatic cleavage to an active intermediate that cross-links DNA via alkylation.
The other agents act directly and do not require in situ activation.
Question 95 of 130
When evaluating a patient with a traumatic hyphema, which of the following is the most likely cause of low intraocular
pressure (IOP)?

Inflammatory cells in the trabecular meshwork

Cyclodialysis cleft

Red blood cells in the trabecular meshwork

Pupillary block
Please select an answer
Feedback:

Blunt trauma may produce a cyclodialysis cleft, which is a separation of the ciliary body from the scleral spur. A
cyclodialysis cleft allows a direct communication between the anterior chamber and suprachoroidal space, and it is
frequently associated with ocular hypotony. A cyclodialysis cleft appears on gonioscopy as a deep angle recession
with a gap between the scleral spur and ciliary body. Ultrasound biomicroscopy can assist in the detection of a
cyclodialysis cleft. Red blood cells and inflammatory cells may obstruct the trabecular meshwork in the setting of a
traumatic hyphema and result in IOP elevation. Rarely, a blood clot in the pupil may produce pupillary block and
elevation of IOP.

Question 96 of 130

Congenital glaucoma is associated with what clinical finding?

Peripheral anterior synechiae

Optic nerve swelling

Tears in Descemet's membrane

Short axial length


Please select an answer
Feedback:

Tears in Descemet's membrane, also known as Haab's striae, are a consequence of buphthalmic enlargement and
stretching of the neonatal globe. Other findings include increased corneal diameter and axial length, optic-nerve
cupping, and scleral thinning. The angle is typically open with a high insertion of the iris root. Peripheral anterior
synechiae are generally not seen, except in children who have had prior angle surgery.
Question 97 of 130
In a sickle cell patient, which agent should be avoided when treating traumatic hyphema?

Beta blockers

Prostaglandin analogues

Carbonic anhydrase inhibitors

Corticosteroids
Please select an answer
Feedback:

Patients with sickle cell hemoglobinopathies have an increased incidence of elevated IOP associated with hyphema
and are more susceptible to complications, including ischemic optic neuropathy. Normal red blood cells (RBCs)
generally pass through the trabecular meshwork without difficulty. However, in sickle-cell hemoglobinopathies
(including sickle trait), the RBCs tend to sickle in the anterior chamber because of the acidity of the stagnant aqueous
humor. These more rigid cells have greater difficulty exiting through the trabecular meshwork and produce increased
IOP. It has been suggested that patients with sickle cell hemoglobinopathies should avoid carbonic anhydrase
inhibitors because these agents may increase the sickling tendency in the anterior chamber by increasing aqueous
acidity. Systemic carbonic anhydrase inhibitors and hyperosmotic agents also have the potential of inducing sickle
crises in susceptible individuals who are significantly dehydrated. Corticosteroids, prostiglandin analogs and beta
blockers are not contraindicated in sickle cell associated hyphema, and are often used in that setting.

Question 98 of 130
Which complication of trabeculectomy is increased with the use of adjunctive anti-fibrotic agents?

Blebitis

Aqueous misdirection

Cystoid macular edema

Encapsulated bleb
Please select an answer
Feedback:

The use of anti-fibrotic agents as adjuncts to trabeculectomy has been shown to increase the risk of bleb-associated
endophthalmitis. Many authors attribute this increase risk to the resultant thin-walled and avascular blebs, which are
seen more often after the use of antifibrotic agents. Other associated risks of antifibtotic agents are hypotony,
increased rates of bleb leaks, and possibly, accelerated cataract formation. Encapsulated blebs, aqueous
misdirection and cystoid macular edema have not been thought to be significantly increased following use of anti-
fibrotic adjuncts.
Question 99 of 130
Which baseline factor was associated with an increased risk of sustained decrease of visual acuity in the Advanced
Glaucoma Intervention Study (AGIS)?

Black race

Older age

Female gender

Worse baseline visual acuity


Please select an answer
Feedback:

The Advanced Glaucoma Intervention Study (AGIS) is a multicenter randomized clinical trial comparing two
sequences of glaucoma surgery, one beginning with argon laser trabeculoplasty (ATT) and the other with
trabeculectomy (TAT). The study explored which baseline risk factors were associated with a sustained decrease of
visual acuity. In both the ATT and TAT sequences, Cox multiple regression analyses identified 3 baseline factors that
were significantly associated with an increased risk of visual acuity loss: better baseline visual acuity, older age at
baseline, and less formal educational level at baseline. Race and gender were not found to be significantly
associated with vision loss in risk factor analyses.

Question 100 of 130

When treating primary infantile glaucoma, at what age is the prognosis highest for successful intraocular pressure control
from trabeculotomy or goniotomy?

Birth

3 years

6 months

10 years

Please select an answer

Feedback:

Angle surgery is less likely to achieve adequate control of intraocular pressure when the symptoms of primary infantile
glaucoma present at birth or after the first year of life. The prognostically favorable window would be between 2
months and 1 year of life.
Question 101 of 130

Which of the following ocular diseases is associated with an increased risk of Wilms tumor?

Peters anomaly

Aniridia

Albinism

Persistent hyperplastic primary vitreous


Please select an answer
Feedback:

Aniridia is a bilateral condition characterized by variable iris hypoplasia. Despite the name and frequent appearance
of a completely absent iris, there is always a rudimentary stump of iris visible on gonioscopy. Most cases of aniridia
are transmitted as an autosomal dominant form, but approximately 1/3 of cases are sporadic. About 20% of cases of
sporadic aniridia are associated with Wilms tumor, with a 67-fold greater chance of developing this tumor compared
with the normal population. However, relatively few cases of Wilms tumor are seen in the familial form of aniridia.
Children with Wilms tumor-aniridia-genitourinary malformation-retardation (WAGR) syndrome are at greatest risk for
developing Wilms tumor. Albinism, Peters anomaly, and persistent hyperplastic primary vitreous are not associated
with a higher risk of Wilms tumor.

Question 102 of 130

Which syndrome is associated with narrowing of the anterior chamber angle?

Pigment dispersion syndrome

Pseudoexfoliation syndrome

Uveitis-hyphema-glaucoma (UGH) syndrome

Sturge-Weber syndrome
Please select an answer
Feedback:

Pseudoexfoliation syndrome is characterized by the deposition of a fibrillar material in the anterior segment of the
eye. Although the origin of this material is not precisely known, it probably arises from multiple sources as part of a
generalized basement membrane disorder. Deposition of pseudoexfoliation material on the lens zonules may lead to
zonular weakness. The anterior chamber angle is often narrow in patients with pseudoexfoliation, presumably as a
result of anterior movement of the lens-iris diaphragm related to zonular weakness. Pigment dispersion syndrome,
UGH syndrome, and Sturge-Weber syndrome generally have an open angle, and narrowing of the anterior chamber
angle is not typically observed in these syndromes.
Question 103 of 130

A patient has uncontrolled intraocular pressure while on maximum tolerated medical therapy following appropriate
laser treatment. Which type of glaucoma would be considered a good candidate for aqueous shunt placement?

Traumatic glaucoma with extensive superior conjunctival scarring

Glaucoma from an intraocular tumor

Neovascular glaucoma with no light perception vision

Primary angle closure glaucoma without previous ocular surgery


Please select an answer
Feedback:

Aqueous shunts traditionally have been reserved for refractory glaucomas in which standard filtering surgery has a
high likelihood of failure. This includes eyes with extensive conjunctival scarring from trauma, previous ocular
surgery, or cicatrizing diseases (eg, ocular cicatricial pemphigoid, Stevens Johnson syndrome). Several secondary
glaucomas have a poor prognosis with trabeculectomy and are good candidates for aqueous shunt placement,
including neovascular glaucoma, uveitic glaucoma, iridocorneal endothelial syndrome, and fibrous/epithelial
ingrowth. However, incisional glaucoma surgery is not indicated in eyes with no light perception vision, and it also
should be avoided in eyes with intraocular tumors because of potential tumor dissemination. A trabeculectomy is
generally the initial glaucoma surgery for low-risk glaucomas such as primary open-angle glaucoma and primary
angle-closure glaucoma.

Question 104 of 130

Which ocular disease is generally characterized by normal intraocular pressure interrupted by recurrent episodes of
markedly elevated intraocular pressure associated with low-grade anterior chamber inflammation?

Fuchs heterochromatic iridocyclitis

Glaucomatocyclitic crisis (Posner-Schlossman syndrome)

Pseudoexfoliation glaucoma

Pigmentary glaucoma
Please select an answer
Feedback:

Glaucomatocyclitic crisis (Posner-Schlossman syndrome) is a rare form of open-angle glaucoma characterized by


recurrent attacks of mild anterior uveitis and marked elevation of intraocular pressure, often to 40-60 mm Hg. The
condition is usually unilateral and affects middle-aged patients. The iritis is generally mild with small, discrete, round
keratitic precipitates. The acute attack usually resolves within a few weeks. The intraocular pressure typically returns
to normal between attacks, but a chronic glaucoma may develop over time with recurrent episodes. Fuchs
heterochromatic iridocyclitis, pseudoexfoliation glaucoma, and pigmentary glaucoma are generally associated with
chronic pressure elevation.
Question 105 of 130

Two months after developing a vitreous hemorrhage, a patient is found to have intraocular pressure of 50 mmHg.
Khaki-colored cells are noted in the anterior chamber. What is the most likely diagnosis?

Schwartz's syndrome

Phacolytic glaucoma

Glaucomatocyclitic crisis (Posner-Schlossman syndrome)

Ghost-cell glaucoma
Please select an answer
Feedback:

Ghost cells are red blood cells that have lost their intracellular hemoglobin and appear as small, khaki-colored
cells. Ghost cells are less pliable than normal red blood cells and obstruct the trabecular meshwork, causing
secondary intraocular pressure elevation, known as ghost-cell glaucoma. The cells develop within 1-3 months
following a vitreous hemorrhage and gain access to the anterior chamber through a disruption of the hyaloid face.
Schwartzs syndrome is a rare secondary open-angle glaucoma that develops in the setting of a rhegmatogenous
retinal detachment in which photoreceptor outer segments migrate through a retinal break to the anterior chamber,
where they impede aqueous outflow. Phacolytic glaucoma is an inflammatory glaucoma caused by leakage of lens
proteins through the capsule of a mature or hypermature cataract, resulting in obstruction of trabecular
outflow. Glaucomatocyclitic crisis (Posner-Schlossman syndrome) is an inflammatory glaucoma characterized by
recurrent attacks of mild anterior uveitis associated with marked intraocular pressure elevation. In the anterior
chamber of Schwartzs syndrome, phacolytic glaucoma, and glaucomatocyclitic crisis are not khaki-colored, the cells
that are seen are not khaki-colored.

Question 106 of 130

Which ocular disease is characterized by the triad of iris heterochromia, iridocyclitis, and cataract?

Fuchs heterochromatic iridocyclitis

Peters anomaly

Glaucomatocyclitic crisis (Posner-Schlossman syndrome)

Iridocorneal endothelial (ICE) syndrome


Please select an answer
Feedback:

Fuchs heterochromatic iridocyclitis is an uncommon form of chronic iridocyclitis characterized by iris heterochromia
with loss of iris pigment in the affected eye, low-grade iritis with stellate keratitic precipitates, and posterior
subcapsular cataract. The condition is usually unilateral and typically affects middle-aged adults. A secondary open-
angle glaucoma develops in approximately 15% of patients. Gonioscopy may reveal fine vessels in the anterior
chamber angle, but these vessels do not lead to peripheral anterior synechiae formation or secondary angle closure.
Cataract and heterochromia are not typical of Posner-Schlossman syndrome. ICE syndrome and Peters anomaly do
not share any features of the triad.
Question 107 of 130

Which multicenter, randomized, clinical trial evaluated the long-term effect of treating patients with newly diagnosed
primary open-angle glaucoma with trabeculectomy versus medical therapy?

Ocular Hypertension Treatment Study (OHTS)

Collaborative Initial Glaucoma Treatment Study (CIGTS)

Early Manifest Glaucoma Trial (EMGT)

European Glaucoma Prevention Study (EGPS)


Please select an answer
Feedback:

The Collaborative Initial Glaucoma Treatment Study (CIGTS) enrolled patients with newly diagnosed primary open-
angle glaucoma and randomized them to initial trabeculectomy or treatment with antiglaucoma medications. After 5
years of follow-up, the rates of visual field progression were similar in the medical treatment group (10.7%) and
surgical treatment group (13.5%). OHTS and EGPS evaluated the efficacy of topical glaucoma medications in
delaying or preventing the onset of primary open-angle glaucoma in patients with ocular hypertension. The EMGT
assessed the efficacy of glaucoma medical and laser therapy in patients with newly diagnosed glaucoma.

Question 108 of 130

A patient with nanophthalmos has persistent appositional-angle closure after a successful iridotomy. Which of the
following laser procedures would be an appropriate treatment to attempt to open the angle?

Cyclophotocoagulation

Laser iridoplasty (gonioplasty)

Laser trabeculoplasty

Capsulotomy
Please select an answer
Feedback:

Laser iridoplasty (gonioplasty) is a procedure in which argon laser burns are applied to the peripheral iris to cause
contraction and flattening. This technique has been shown to be useful in opening an angle that remains narrow after
an iridotomy, as with plateau iris syndrome and nanophthalmos. Laser trabeculoplasty, cyclophotocoagulation, and
capsulotomy would not be helpful in opening a narrow angle.
Question 109 of 130

What glaucoma medication lowers intraocular pressure mainly by reducing the rate of aqueous production?

Latanoprost

Travaprost

Pilocarpine

Timolol
Please select an answer
Feedback:

Commercially available glaucoma medications lower intraocular pressure by either decreasing the rate of aqueous
humor production or increasing the outflow of aqueous through the trabecular or uveoscleral pathways. Beta blockers
such as timolol inhibit cyclic adenosine monophosphate (cAMP) production in the ciliary epithelium, thereby reducing
aqueous humor secretion and lowering intraocular pressure. Pilocarpine causes contraction of the ciliary muscle,
which pulls on the sclera spur to open the trabecular meshwork, reducing intraocular pressure because of increased
trabecular outflow. Prostaglandin analogues such as latanoprost and travoprost mainly lower intraocular pressure by
increasing uveoscleral outflow.

Question 110 of 130

Which of the following ocular conditions would most likely be associated with ocular hypotony?

Peripheral anterior synechiae

Ocular ischemia

Aqueous misdirection

Angle recession

Please select an answer

Feedback:

Ocular hypotony, or low intraocular pressure, is frequently observed in the setting of ocular ischemia. Decreased
perfusion of the ciliary body results in a reduced rate of aqueous humor production and a low intraocular
pressure. Other causes of ocular hypotony include wound leaks, ocular inflammation, cyclodialysis cleft,
cyclodestruction, overfiltration after glaucoma surgery, glaucoma medical therapy, and retinal detachment. Angle
recession, peripheral anterior synechiae, and aqueous misdirection are conditions that are generally associated with
elevation of intraocular pressure, but not ocular hypotony.

Question 111 of 130

What structural abnormality may produce ocular hypotony?

Tenon's cyst

cyclodialysis cleft

Angle recession

Elevated episcleral venous pressure


Please select an answer
Feedback: Hypotony refers to a decrease in intraocular pressure below the normal physiologic range. Hypotony may
result from an underproduction of aqueous humor or excessive aqueous outflow. Causes of ocular hypotony include
a cyclodialysis cleft, overfiltration after glaucoma surgery, retinal detachment, choroidal detachment, iritis, would leak,
and ocular ischemia. Angle recession, elevated episcleral venous pressure, and a tenon's cyst may be associated
with an increase in intraocular pressure, but not ocular hypotony.

Question 112 of 130

Argon laser treatment produces a white burn when applied to an abnormally smooth iris. This response is
characteristic of what ocular condition?

Iridocorneal endothelial (ICE) syndrome

Iris melanoma

Epithelial downgrowth

Persistent fetal vasculature


Please select an answer
Feedback:

Epithelial downgrowth is an uncommon condition in which epithelium invades into the anterior chamber through a
surgical wound or penetrating injury. Epithelial downgrowth presents as a grayish, sheet-like membrane on the
trabecular meshwork, iris, ciliary body, and posterior surface of the cornea. Argon laser produces a characteristic
white burn when applied to the epithelial membrane on the iris surface. This typical reaction can be helpful in
confirming the diagnosis and identifying the extent of involvement.

Iris membranes associated with ICE syndrome and persistent fetal vasculature do not create a white burn as there is
little tissue to coagulate on the iris. Iris melanomas do not cause a smoothing of the iris surface.
Question 113 of 130
A 14-year-old boy with bilateral iris atrophy and corectopia is found to have elevated intraocular pressures. His father
has a similar condition. Which of the following is the most likely diagnosis?

Iridocorneal endothelial syndrome

Lowe syndrome

Axenfeld-Rieger syndrome

Hallermann-Streiff syndrome
Please select an answer
Feedback: Axenfeld-Rieger syndrome is a group of bilateral congenital anomalies involving anterior segment
structures of neural crest origin. Approximately 50% of cases are associated with glaucoma. Although this syndrome
was initially separated into Axenfeld anomaly (posterior embryotoxon with multiple iris processes), Rieger anomaly
(Axenfeld anomaly plus iris hypoplasia and corectopia), and Rieger syndrome (Rieger anomaly plus developmental
defects of the teeth or facial bones, redundant periumbilical skin, pituitary abnormalities, or hypospadius), these
disorders are now considered variations along the same clinical spectrum and are combined under the name
Axenfeld-Rieger syndrome. Iridocorneal endothelial (ICE) syndrome may have a similar appearance with iris atrophy
and corectopia, but this condition is unilateral and not inherited. Lowe syndrome and Hallerman-Streiff syndrome are
systemic congenital disorders that are commonly associated with glaucoma, however, they do not have iris atrophy
and corectopia.

Question 114 of 130


What anatomical modification is found in increased frequency in primary congenital glaucoma?

Hypoplastic optic nerve

Increased axial length

Hyperopia

Decreased corneal diameter


Please select an answer
Feedback: Primary congenital glaucoma is caused by an abnormal development of the anterior chamber angle. It is
a rare disease occurring in 1 in 10,000 births, but it accounts for approximately 50-70% of congenital glaucoma
cases. During the first 3 years of life, the collagen fibers of the eye are more elastic than in older individuals. As a
result, elevation of intraocular pressure causes rapid enlargement of the globe producing an increase in axial length
and corneal diameter as a typical clinical manifestation of this disease. Optic nerve cupping and pallor are generally
seen, but optic disc hypoplasia is not a characteristic feature of congenital glaucoma. An increase in axial length
usually induces myopia, not hyperopia.

Question 116 of 130


What condition is associated with both ectopia lentis and pupillary block?

Weill-Marchesani syndrome

Fuchs heterochromatic iridocyclitis

Axenfeld-Rieger syndrome

Iridocorneal endothelial (ICE) syndrome


Please select an answer
Feedback:

Ectopia lentis refers to displacement of the lens from its normal anatomic position. Weill-Marchesani is a condition
associated with microspherophakia (a congential disorder in which the lens has a spherical or globular shape) and
ectopia lentis. As a result, patients with Weill-Marchesani are at increased risk of angle-closure glaucoma from
pupillary block.

Fuchs heterochromic iridocyclitis and Axenfeld-Reiger and ICE syndromes are associated with open angle
glaucomas.

Most other conditions that are associated with ectopia lentis such as Marfan syndrome and trauma, are not
associeated with angle closure glaucoma rather than open angle or angle recession glaucomas.

Question 117 of 130


Which of the following may cause erroneously low measurement of intraocular pressure by Goldmann applanation
tonometry?

Decreased central corneal thickness

Excessive fluoroescein

Squeezing of the eyelids

Pressure on the globe


Please select an answer
Feedback: The Goldmann applanation tonometer measures the force necessary to flatten an area of the cornea that
is 3.06 mm in diameter. It is based on the Imbert-Fick principle, which states that the pressure inside an ideal sphere
equals the force necessary to flatten its surface divided by the area of flattening. There ar multiple possible sources
of error in tonometry. Squeezing the eyelids and pressure on the globe can falsely elevate IOP. Excessive
fluoroscein results in wide mires and an inaccurately high reading, whereas inadequate fluoroscein leads to low
readings. Increased central corneal thickness may produce artificially high IOP measurement, and decreased central
corneal thickness may give an artificially low IOP.

Question 118 of 130


Which class of glaucoma medications should be avoided in a patient with a sulfa allergy?

Carbonic anhydrase inhibitors

Prostaglandin analogues

Miotics

Beta blockers
Please select an answer
Feedback: Carbonic anhydrase inhibitors decrease aqueous humor formation by inhibiting carbonic anhydrase in the
ciliary epithelium. Carbonic anhydrase inhibitors are frequently used orally (acetazolamide and methazolamide) and
topically (dorzolamide and brinzolamide) in the treatment of glaucoma. Carbonic anhydrase inhibitors are chemically
derived from sulfa drugs, and they may cross react with sulfa drugs. The other classes of glaucoma medications are
not sulfa derivatives, and no cross reactivity with sulfa drugs is expected.
Question 119 of 130
For what pathological process does argon laser application applied to the iris/iris membranes produce white burns?

neovascularization

iris nevus syndrome

posterior polymorphous dystrophy

epithelial downgrowth
Please select an answer
Feedback: Epithelial downgrowth occurs when epithelium invades the anterior chamber through a surgical or
traumatic wound. Epithelial ingrowth presents as a grayish, sheet-like growth on the posterior cornea, iris, trabecular
meshwork, and ciliary body. Argon laser treatment produces characteristic white burns of the epithelial membrane on
the iris surface, which can be helpful to confirm the diagnosis and determine the extent of involvement. Epithelial
downgrowth can cause a refractory secondary glaucoma. Radical surgery is sometimes recommended to remove
the intraocular epithelial membrane and affected tissue and repair the fistula, but the prognosis is generally poor.

Question 120 of 130

What is the most common inheritance pattern for Axenfeld-Rieger syndrome?

Sporadic

X-linked

Autosomal dominant

Autosomal recessive

Please select an answer

Feedback: Axenfeld-Rieger syndrome is a group of bilateral congenital anomalies that may include abnormal
development of the anterior chamber angle, iris, and trabecular meshwork. Autosomal dominant inheritance occurs
in most cases, but Axenfeld-Rieger syndrome is occasionally sporadic. About 50% of cases are associated with
glaucoma. The typical corneal abnormality is a posterior embryotoxon (a prominent anteriorly displaced Schwalbe's
line). Iris processes are characteristically seen extending to Schwalbe's line. The iris may range from normal to
markedly abnormal with corectopia, hole formation, and ectropion uvae. Developmental defects of the teeth or frontal
bones may occur in conjunction with the ocular abnormalities.
Question 121 of 130
Which visual field testing method uses low spacial frequency sinusoidal gratings that undergo rapid phase-reversal
flicker?

Short-wavelength automated perimetry (SWAP)

Frequency-doubling technology (FDT) perimetry

Suprathreshold testing

Swedish interactive thresholding algorithm (SITA)


Please select an answer
Feedback:

Frequency-doubling technology (FDT) uses a low spacial frequency sinusoidal gratings that undergo rapid phase-
reversal flicker. When gratings are presented in this way, there appears to be twice as many alternating light and
dark bars than are actually present (hence the term frequency doubling). It is believed that the stimuli employed in
this test preferentially activate M cells and may be more sensitive in the detection of early glaucoma.

Short-wavelength authomated perimetry does not use grating as stimuli, but rather blue stimuli on a yellow
background. Suprathreshold testing uses white-on-white stimuli to screen for dense scotoma. Swedish interactive
thresholding algorithm (SITA) is an algorighm that can be applied to any stimulus/background paradigm to estimate
target thresholds of adjacent testing points to speed testing.

Question 122 of 130


What abnormality of the optic disc rim is not specific for early glaucomatous damage?

optic disc rim pallor

asymmetric appearance of the optic disc or rim between the two eyes

diffuse or focal narrowing of the disc rim, especially at the inferior and superior poles

optic disc hemorrhage


Please select an answer
Feedback: Glaucomatous optic neuropathy is the sine qua non of all forms of glaucoma. Careful examination of the
optic disc is critical in detecting glaucoma. Diffuse or focal narrowing of the disc rim may occur with glaucomatous
optic nerve damage. Focal atrophy most typically occurs at the inferior and superior poles of the optic nerve in early
glaucoma, which may produce vertical elongation of the optic nerve cup and/or a focal notch. Asymmetry of the cup-
disc ratio may also be an early sign of glaucoma, with cup-disc asymmetry of more than 0.2 occuring in less than 1%
of normal individuals. Optic disc hemorrhages generally occur in the nerve fiber layer at the neuroretinal rim or in the
peripapillary area, and are associated with glaucoma (especially normal tension glaucoma). Optic disc pallor is seen
in many types of optic neuropathies and is not specific for glaucoma.
Question 123 of 130
During automated static perimetry, a patient responds when no stimulus is presented. What type of error is this?

short-term fluctuation

false negative error

fixation loss

false positive error


Please select an answer
Feedback: Automated static perimetry determines the threshold sensitivities at multiple points in the visual field.
Whe a patient responds at a time when no test stimulus is presented, a false positive error is recorded. Whe a
patient fails to respond to a stimulus presented in a location where a dimmer stimulus was previously seen, a false-
negative response is recorded. Fixation losses are identified when the patient fails to look at the central fixation light
during testing. Short-term fluctuation is a measure of normal physiologic variation and intratest reliability, and it is
measured when the visual field analyzer double-determines 10 preselected points during the couse of the test
session.

Question 124 of 130


The Tube Versus Trabeculectomy (TVT) Study is a multicenter randomized clinical trial comparing the safety and
efficacy of tube shunt surgery and trabeculectomy with mitomycin C (MMC) in patients who had previous cataract
and/or glaucoma surgery. Which glaucoma procedure had a higher rate of surgical failure in this study?

The TVT Study did not report surgical failure rates

Trabeculectomy with MMC had a higher failure rate

Tube shunt surgery had a higher failure rate

No significant difference in failure rates was observed in the study


Please select an answer
Feedback: The TVT Study enrolled patients with medically uncontrolled glaucoma who had previous cataract
extraction with intraocular lens implantation and/or failed filtering surgery and randomized them to receive a 350-mm2
Baerveldt glaucoma implant or trabeculectomy with MMC. Failure was defined as IOP > 21 mm Hg or not reduced by
20% below baseline, IOP 5 mm Hg, reoperation for glaucoma, or loss of light perception vision. The cumulative
probability of failure during 3 years of follow-up was 15.1% in the tube group and 30.7% in the trabeculectomy group,
a difference that was statistically significant (p = 0.010). Early postoperative complications were more common after
trabeculectomy with MMC than tube shunt surgery. However, both procedures had similar rates of late postoperative
complications and serious complications.
Question 125 of 130
The nerve fiber layer is best visualized at the slit lamp using what special technique?

red-free illumination

specular reflection

fluorescein angiography

cobalt blue illumination


Please select an answer
Feedback:

Careful ophthalmoscopic examination of the nerve fiber layer may be helpful in diagnosing glaucoma, and the nerve
fiber layer is best visualized with red free (green) illumination. The nerve fiber layer extending from the neuroretinal
rim to the surrounding peripapillary retina appear as fine striations created by bundles of retinal ganglion cell axons.
In the normal eye, the nerve fiber layer has a plush, refractile appearance. With progressing glaucomatous optic
neuropathy, the nerve fiber layer becomes less visible. Nerve fiber loss may be focal or diffuse. Focal abnormalities
of the nerve fiber layer appear as wedge defects extending to the optic nerve. Diffuse nerve fiber layer loss is more
difficult to detect than focal loss.Cobalt blue illumination and fluorescein angiography do not enhance visualization of
the nerve fiber layer. Specular reflection is useful for visualizing abnormalities on the posterior corneal surface but is
not achievable in the posterior segment.

Question 126 of 130

Which of the following has been reported to cause secondary angle-closure glaucoma secondary to congestion of the
ciliary body and ciliochoroidal detachment?

azithromycin(Zithromax)

acetaminophen(Tylenol)

valium

topiramate (Topamax)
Please select an answer
Feedback: Topiramate (Topamax) is an oral sulfa medication that is used to treat epilepsy, migraines, and
depression. There have been reports of acute bilateral angle-closure glaucoma associated with this medication.
Ocular findings of this syndrome include axial shallowing of the anterior chamber with a forward shift of the lens-iris
diaphragm, induced myopia, markedly elevated intraocular pressure, and a closed angle. The syndrome appears to
result from swelling of the ciliary body with the development of a ciliochoroidal effusion/detachment allowing the lens-
iris diaphragm to shift foward. Azithromycin, valium, and acetaminophen have not been reported to produce
secondary angle-closure glaucoma.
Question 127 of 130
The parents of a 4-month-old boy noted that their son is unusually sensitive to light and has tearing in both eyes.
They have also noted that their child has developed "large eyes" and they are unable to see his pupils. What is the
most likely diagnosis?

Nasolacrimal duct obstruction

Primary congenital glaucoma

Cystinosis

Congenital hereditary endothelial dystrophy


Please select an answer
Feedback: Primary congenital glaucoma accounts for about 50-70% of all congenital glaucoma. Approximately 65%
of patients with this condition are male, and 70% have bilateral involvement. The diagnosis of primary congenital
glaucoma is made during the first year of life in 80% of cases. Primary congenital glaucoma classically presents with
the triad of epiphora, photophobia, and blepharospasm in an infant. The infant eye enlarges in response to elevated
IOP producing buphthalmos. Tears in Descemet's membrane, called Haab striae, may occur from corneal
stretching. Corneal edema may develop in association with Haab striae and IOP elevation. Nasolacrimal duct
obstruction typically produces epiphora, but does not explain the findings of corneal edema and buphthalmos in this
infant. Congenital hereditary endothelial dystrophy and cystinosis can produce corneal clouding at birth, but not
buphthalmos.

Question 128 of 130


A 69-year-old woman has been followed for anatomically narrow angles and has previously undergone laser
iridotomies in both eyes. During a preoperative evaluation for cataract surgery, her axial length is measured as 19.5
mm in each eye. Which of the following procedures should be performed in conjunction with her cataract extraction?

Descemet's stripping automated endothelial keratoplasty

Pars plana vitrectomy

Posterior sclerotomies

Trabeculectomy
Please select an answer
Feedback: Nanophthalmos is a condition in which the eye is normal in shape but unusually small in size, with an
axial length less than 20 mm. Patients with nanophthalmos generally have thickened sclera that may impede
drainage from the vortex veins, and intraocular surgery is frequently complicated by choroidal effusion and
nonrhegmatogenous retinal detachment. Patients with nanophthalmos frequently have narrow angles that are best
managed with laser iridotomy, and laser iridoplasty in select cases. When intraocular surgery is performed, it is
prudent to place prophylactic posterior sclerotomies to reduce the risk of intraoperative and postoperative choroidal
effusion.
Question 129 of 130
What is the estimated prevalence of ocular hypertension in the population older than age 40 years in the United
States?

15-25%

4-7%

1-2%

25-35%
Please select an answer
Feedback: Ocular hypertension is a condition in which intraocular pressure is elevated in the absence of identifiable
optic nerve damage or visual field loss. The prevalence of ocular hypertension in the population older than age 40
years in the United States has been estimated to range from 4% to 7%. Elevated intraocular pressure is a risk factor
for the subsequent development of glaucoma. However, many individuals with ocular hypertension never develop
glaucoma.

Question 130 of 130

What surgical procedure that is used to manage congenital glaucoma involves surgery from an ab interno approach?

Aqueous shunt implantation

Trabeculectomy

Trabeculotomy

Goniotomy

Please select an answer

Feedback: Goniotomy is a procedure that is used to treat congenital glaucoma, and the operation is performed from
an ab interno or anterior chamber approach. Goniotomy involves the passage of a needle-knife through a distant
corneal incision, across the anterior chamber, making a superficial incision in the anterior aspect of the trabecular
meshwork under gonioscopic guidance. A clear cornea is necessary to provide adequate visualization to identify
landmarks needed to perform a goniotomy. Trabeculotomy is another procedure that is commonly used to treat
congenital glaucoma, but this operation is performed by an ab externo approach. Trabeculotomy involves inserting a
trabectome into Schlemm's canal from an external incision, and the trabectome is then rotated into the anterior
chamber tearing the trabecular meshwork. Trabeculectomy and aqueous shunt implantation are also procedures that
are performed fron an external approach, rather than an internal approach.

Collected from www.aao.org (self assessment)


By Dr. AlBaraa AlQassimi

You might also like